Care of Women KSA 2023

Lakukan tugas rumah & ujian kamu dengan baik sekarang menggunakan Quizwiz!

A 24-year-old graduate student comes to your office to be tested for sexually transmitted infections. The medical assistant tells you that the patient was upset when she saw how much she weighed. On questioning, the patient says that for the past year she has experienced episodes of uncontrollable eating followed by self-induced vomiting. Her weight is 82 kg (181 lb) and her BMI is 32 kg/m2. Which one of the following is true regarding treatment for this condition? A. Cognitive behavioral therapy has the best evidence for treatment B. SSRI monotherapy is a first-line treatment option C. Anemia is an indication for hospitalization D. More than half of patients will relapse after treatment

ANSWER: A Eating disorders include anorexia nervosa, bulimia nervosa, and binge eating disorder, and the DSM-5 added avoidant/restrictive food intake disorder, rumination disorder, and pica to this group in 2014. Mood disorders, anxiety, substance use, and personality or somatic disorders are common in these patients. Screening can include regularly asking questions about mood, body image concerns, and eating behaviors. Before establishing the diagnosis based on history, it is important to perform a physical examination that includes measurement of orthostatic vital signs and obtain a metabolic panel that includes magnesium and phosphate levels. This patient appears to have bulimia nervosa, which consists of eating an excessive amount of food in a short period of time (often >2000 calories in one sitting), with a concomitant feeling of loss of control. Because patients with bulimia base their self-worth on their body shape and weight, they follow this binge eating with compensatory behaviors to prevent weight gain, such as vomiting, laxative use, food restriction, excessive exercise, or taking diuretics. Episodes occur, on average, one or more times a week for 3 months or longer, and the disorder is associated with a two- to sixfold increase in age-adjusted mortality. After the diagnosis is established and a goal weight has been accepted, the patient is best served with treatment delivered by a team that includes a therapist, a nutritionist, and a clinician, preferably with each having prior experience in caring for patients with eating disorders. Cognitive behavioral therapy (CBT) has the best evidence for treatment of adults with bulimia, while family-based therapy is the first-line treatment for adolescents with this condition. Early behavioral response, with rapidly declining episodes of binge eating, is associated with a greater chance of sustained remission.Medication should not be considered as monotherapy for eating disorders, and care must be taken to avoid medications that affect electrolyte levels or heart rate, or predispose the patient to a prolonged QT interval. Fluoxetine titrated up to 60 mg daily is FDA approved for the treatment of bulimia, and may be used as an adjunct to CBT. Other SSRIs have demonstrated benefit as well, but all must be prescribed at dosages higher than those used to treat depression. Citalopram and escitalopram should be avoided because of the increased risk of prolonging the QT interval. Criteria for inpatient treatment of bulimia include dehydration, hematemesis, syncope or seizure, EKG abnormalities, autonomic dysfunction (bradycardia, hypotension, hypothermia, orthostatic blood pressure), electrolyte abnormalities, and uncontrollable symptoms or co-occurring conditions that cannot be treated successfully on an outpatient basis. Disordered eating may persist for as long as 20 years after the diagnosis is made in approximately one-third of patients. Success rates for treatment of bulimia are higher than those for anorexia nervosa, with less than one-third of patients experiencing relapse after treatment.

You see a 27-year-old gravida 2 para 2 on her second postpartum day after a normal vaginal delivery. On examination she has a temperature of 38.2°C (100.8°F), but it was as high as 38.6°C (101.5°F) in the first 24 hours after delivery. She has no respiratory or urinary symptoms. Her uterus is mildly tender. She has no allergies to medications. Which one of the following intravenous antibiotic regimens would be appropriate for this patient? A. Ampicillin plus gentamicin B. Clindamycin (Cleocin) plus gentamicin C. Cefazolin D. Cefoxitin

ANSWER: B Postpartum endometritis (or puerperal fever) may be diagnosed in a febrile postpartum patient who has no evidence of another infection, such as a urinary tract infection or pneumonia. Uterine tenderness, foul smelling lochia, and a high WBC count support the diagnosis, but are not essential for making the diagnosis. Cultures are not necessary since the usual etiology is polybacterial, including anaerobes. The standard definition used for puerperal fever is an oral temperature of at least 38.0°C (100.4°F) on any 2 of the first 10 days post partum, or 38.7°C (101.7°F) in the first 24 hours post partum. Typical rates of postpartum endometritis are 1%-3% after vaginal birth and 10%-30% after cesarean section. In addition to cesarean delivery, risk factors include duration of labor, number of vaginal examinations, duration of ruptured membranes prior to delivery, and anemia. A Cochrane review demonstrated that antibiotic prophylaxis at the time of cesarean section reduces the risk of postpartum endometritis by about 60%. Ampicillin and first-generation cephalosporins are equally effective (SOR A). Using multiple antibiotics for prophylaxis is not more effective. Another Cochrane review showed that intravenous clindamycin + gentamicin, the traditional gold-standard therapy, is appropriate because no other regimen has been shown to be more effective. When compared to treatment with cephalosporins, clindamycin + gentamicin resulted in fewer treatment failures. Clindamycin + gentamicin once daily has been shown to be as effective as the usual three daily doses. If another antibiotic regimen is chosen, it should be one with activity against penicillin-resistant anaerobes such as Bacteroides fragilis (SOR A). Among the regimens listed, neither ampicillin plus gentamicin nor cefazolin has activity against beta-lactam-resistant anaerobes. After clinical improvement has been observed with intravenous antibiotics, additional oral antibiotics are not needed, as they have not been proven to be beneficial.

A 23-year-old female presents to your office with concerns about irregular menstrual periods occurring every 2-3 months. Her blood pressure is 138/82 mm Hg, her pulse rate is 66 beats/min, and her BMI is 32 kg/m2. On examination you note coarse, dark hair on her upper lip, chin, abdomen, and upper and inner thighs, and she has moderate inflammatory acne on her face and upper back. The remainder of the examination is normal. As you consider a diagnosis of polycystic ovary syndrome, which one of the following conditions should be ruled out initially? A. An androgen-secreting tumor B. Cushing syndrome C. HAIR-AN syndrome (severe insulin resistance) D. Hypothyroidism E. Primary ovarian insufficiency

ANSWER: D Making a diagnosis of polycystic ovary syndrome (PCOS) requires the systematic exclusion of conditions that may mimic this syndrome. In women with evidence of ovulatory dysfunction, other causes such as thyroid dysfunction and hyperprolactinemia should be excluded. Because of the wide range of manifestations of thyroid disease and its high frequency in women with menstrual disorders, a TSH level should be checked in women with possible PCOS. A prolactin level should also be obtained to rule out hyperprolactinemia as a cause of anovulation in women with suspected PCOS. HAIR-AN syndrome, a rare subphenotype of PCOS, consists of hyperandrogenism, severe insulin resistance, and acanthosis nigricans, and occurs in nearly 5% of women with hyperandrogenism. An androgen-secreting tumor is characterized by a rapid onset of virilization symptoms, including changes in voice, male pattern androgenic balding, and clitoromegaly. Testing is not indicated in the absence of these symptoms. Primary ovarian insufficiency involves amenorrhea (as opposed to oligomenorrhea) combined with symptoms of estrogen deficiency, including hot flashes or urogenital symptoms. The patient does not have any of these symptoms so testing for this would not be appropriate at this point. Because Cushing syndrome is extremely rare (1 in 1,000,000 individuals) and screening tests are not 100% sensitive or specific, routine screening for Cushing syndrome in all women with hyperandrogenic chronic anovulation is not indicated. Those who have coexisting signs of Cushing syndrome, including moon facies, abdominal striae, centripetal fat distribution, or hypertension, should be considered for screening with a 24-hour free cortisol level or a dexamethasone suppression test.

A 44-year-old premenopausal female sees you to discuss chemoprevention after a breast biopsy reveals atypical ductal hyperplasia. She has normal menstrual cycles and her family history is notable for invasive breast cancer in a sister at age 48. After a conversation about risks and benefits of chemoprevention and possible side effects she decides to start medication to reduce her risk. Which one of the following would you recommend for this patient? A. Combined oral contraceptives B. Letrozole (Femara) C. Medroxyprogesterone (Provera) D. Raloxifene (Evista) E. Tamoxifen (Soltamox)

ANSWER: E The U.S. Preventive Services Task Force (USPSTF) recommends offering medications that have been shown to reduce the risk of breast cancer to any woman at increased risk for breast cancer who is also at low risk for adverse medication effects (B recommendation). These medications include tamoxifen, raloxifene, and aromatase inhibitors. These drugs provide at least a moderate benefit in reducing estrogen-receptor-positive breast cancer in postmenopausal women. However, the USPSTF recommends against routinely using these medications in women with no increased risk for breast cancer (D recommendation). Tamoxifen, a systemic estrogen reuptake inhibitor, was approved by the FDA in 1998 for primary prevention of breast cancer in high-risk women. It can decrease the relative risk of developing breast cancer (specifically estrogen-receptor-positive breast cancer) by up to 48%. It is the only FDA-approved medication for the chemoprevention of breast cancer in premenopausal women (SOR A). Raloxifene is another selective estrogen receptor modulator and is approved for the chemoprevention of breast cancer in postmenopausal women, but not premenopausal women (SOR A). Tamoxifen is therefore a better choice in this patient. Both tamoxifen and raloxifene have been shown to also reduce the risk of skeletal fractures. Letrozole is an aromatase inhibitor. Aromatase inhibitors are approved for chemoprevention of breast cancer in postmenopausal women (SOR A). They block the conversion of androgens to estrogen but cannot block ovarian production of estrogen, so they do not work in premenopausal women unless the woman is also taking a GnRH inhibitor. Combined oral contraceptives can be used for the prevention of ovarian cancer but do not prevent breast cancer (SOR C). Progesterone does not reduce the risk of breast cancer (SOR A).

A 28-year-old patient presents with crampy left-sided pelvic pain that began last night. She had a positive pregnancy test a week ago but has not had any bleeding, fever, or emesis. Her only previous pregnancy resulted in a miscarriage 4 years ago. She is very anxious about these current symptoms. She is uncertain when her last menstrual period started but she thinks it was about 6 weeks ago. On examination you confirm diffuse tenderness in the mid- and left-sided pelvic area with no peritoneal signs, no bleeding on speculum examination, and no apparent cervical or vaginal abnormalities. You order transvaginal ultrasonography, which shows no sign of intrauterine pregnancy and no adnexal mass. Her quantitative β-hCG level is 900 mIU/mL. Which one of the following is true regarding this situation? A. The negative pelvic ultrasonography rules out an ectopic pregnancy B. The quantitative β-hCG rules out an ectopic pregnancy C. The combination of the ultrasound findings and the β-hCG level is diagnostic of an anembryonic pregnancy (blighted ovum) D. Dilatation and curettage is indicated to distinguish between an anembryonic pregnancy and an ectopic pregnancy E. She may still have a normal pregnancy

ANSWER: E With a quantitative β-hCG level <1500 mIU/mL and no evidence of intrauterine or ectopic pregnancy on ultrasonography, neither anembryonic pregnancy nor ectopic pregnancy can be ruled out. This patient may have a normal pregnancy, have an ectopic pregnancy, or miscarry. Therefore, it is too early to consider any invasive procedures that would put the pregnancy at risk. When the β-hCG level is in the 1500-3000 mIU/mL range (referred to as the discriminatory level), a gestational sac should be seen on transvaginal ultrasonography. Failure to detect an intrauterine pregnancy on ultrasonography when the β-hCG level is in this range is concerning for early pregnancy loss or ectopic pregnancy. When the gestational sac is 10 mm in size, a yolk sac should be seen, which confirms an intrauterine pregnancy. Occasionally, a woman may have an intrauterine AND an ectopic pregnancy. This is referred to as a heterotopic pregnancy and occurs in approximately 1 in 30,000 pregnancies. When ultrasound findings are not definitive, the location of the pregnancy cannot be established. No single measurement of β-hCG, regardless of its value, can reliably distinguish between an ectopic and an intrauterine pregnancy. If a single β-hCG measurement is <3500 mIU/mL, presumptive treatment for ectopic pregnancy, either methotrexate or surgery, should not be undertaken, given the risk of interrupting a viable intrauterine pregnancy. As long as the patient is stable, repeat β-hCG testing and transvaginal ultrasonography are warranted before making a diagnosis of ectopic pregnancy.

An 18-year-old female sees you for evaluation of heavy menstrual bleeding that has been present since menarche. She reports that her periods have always been regular, occurring every 28 days, but they last 8-10 days and necessitate pad changes every 1-2 hours. On examination her vital signs and weight are normal, there is no bruising or petechiae, and no abnormalities are noted. An in-office pregnancy test is negative and a CBC has been ordered. Of the following underlying etiologies, which one is most likely in this patient? A. A bleeding disorder B. Endometrial hyperplasia C. Hyperprolactinemia D. Polycystic ovary syndrome

ANSWER: A Abnormal uterine bleeding can be categorized as ovulatory (regular cycles) or anovulatory (irregular cycles). This patient has ovulatory abnormal uterine bleeding (menorrhagia) evidenced by heavy bleeding that lasts for more than 7 days and occurs at regular intervals every 24-35 days. Up to 20% of women presenting with heavy menstrual bleeding will have an underlying inherited bleeding disorder, with a higher prevalence in adolescent females. The onset of heavy menses at menarche is often the first sign. An initial evaluation for a bleeding disorder is indicated when a patient reports heavy menstrual bleeding since menarche, postpartum hemorrhage, surgery-related bleeding, bleeding associated with dental work, or two or more of the following: bruising 1-2 times per month, epistaxis 1-2 times per month, frequent gum bleeding, or a family history of bleeding symptoms. Heavy menses may be seen with platelet disorders and factor deficiencies (factors VIII and IX, and less commonly VII and XI). The initial evaluation should include a prothrombin time and partial thromboplastin time when a bleeding disorder is suspected. Results may be normal in women with von Willebrand disease, and other testing may be appropriate, including plasma von Willebrand antigen, a von Willebrand factor-ristocetin cofactor assay, and factor VIII. Polycystic ovary syndrome can cause heavy bleeding, but it is associated with anovulatory bleeding characterized by irregular menstrual cycles. Recurrent anovulation leads to endometrial hyperplasia. This patient is not at high risk for endometrial hyperplasia due to her age, history of regular cycles, and lack of risk factors. Hyperprolactinemia usually leads to anovulation, which would be characterized by irregular or absent menstrual cycles.

A 28-year-old gravida 1 para 1 presents with a 6-month history of bilateral spontaneous nipple discharge, which she describes as milky and without any apparent blood. Her pregnancy was 4 years ago. She has not had any breast pain and there is no history of trauma. A review of systems is negative except for a 1-year history of oligomenorrhea. Her past medical history is significant for bipolar disorder. Her current medications include divalproex sodium (Depakote) and risperidone (Risperdal). On examination the patient's vital signs are normal. A thin, milky, guaiac-negative discharge is expressed from both nipples. There are no breast masses or adenopathy. A neurologic examination is also normal. A pregnancy test is negative, her TSH level is 2.8 µU/mL (N 0.4-4.2), and her serum prolactin level is 100 µg/L (N 4-23). Which one of the following would be the next appropriate step in evaluating this patient? A. Substitution of aripiprazole (Abilify) for risperidone B. Cytology of the nipple discharge C. Mammography D. Brain MRI

ANSWER: A Nipple discharge can be physiologic or pathologic, with the predominant causes being physiologic. Common causes of physiologic galactorrhea include medication-induced hyperprolactinemia, pituitary tumors, hypothyroidism, renal insufficiency, pregnancy, and nipple stimulation. The initial workup should include a pregnancy test and thyroid function testing. With hypothyroidism, increased thyrotropin-releasing hormone levels can stimulate prolactin secretion. A careful review of the patient's medications may also reveal iatrogenic causes of the nipple discharge. This patient's hyperprolactinemia and subsequent galactorrhea are most likely due to the risperidone, which has a high frequency of associated prolactin elevation. Dopamine inhibits prolactin secretion and dopamine antagonists can lead to elevated prolactin secretion. Some antipsychotics (risperidone, paliperidone, and the high-potency, first-generation antipsychotics) are most commonly associated with this side effect. The recommended management is to change the risperidone to another medication that is known to be effective for treating bipolar disorder and repeat the prolactin level after a minimum of 3 days. Aripiprazole would be the preferred option, given that it is a closer agent to risperidone and has been shown to reduce the incidence of galactorrhea in these patients. In the double-blind DAAMSEL (Dopamine Partial Agonist, Aripiprazole, for the Management of Symptomatic Elevated Prolactin) clinical trial, aripiprazole was shown to be particularly effective at improving galactorrhea, with almost 80% of women having resolution of this side effect at the end of the 16-week trial. Pathologic nipple discharge, characterized by a spontaneous unilateral, uniductal discharge with bloody features, should also be ruled out. Ultrasonography is the first-line modality in women <30 years of age and mammography is the first-line modality in women ≥30 years of age. MRI of the brain is not indicated unless the cause of the prolactin elevation cannot be found on the initial workup. Cytology of the nipple discharge is used to identify a cause of pathologic discharge in women with negative findings on ultrasonography or mammography. Its use is limited because of its high rate of negative findings.

A 23-year-old female presents with nausea and dull left adnexal pain that started 48 hours ago. She has no vaginal discharge or urinary or gastrointestinal symptoms. She is monogamous with a male sexual partner and reports that they consistently use condoms during intercourse. Her last menstrual period began 31 days ago. She was seen in the emergency department last evening and a serum pregnancy test was negative. Transvaginal ultrasonography showed a unilocular, thin-walled, anechoic lesion measuring 45 mm on her left ovary. She took over-the-counter naproxen (Aleve), 440 mg, for pain last night before she went to bed. She recalls a similar episode of pain last year that resolved over a couple of weeks, but she is not sure whether it was on the left or right side. Which one of the following would be appropriate at this point in the management of this patient's ovarian cyst? A. Monitoring to see if the cyst will resolve spontaneously B. Repeat ultrasonography in 2 weeks C. Oral contraceptive pills to hasten the resolution of the cyst D. Immediate laparoscopy to rule out malignancy

ANSWER: A Ovarian cysts are common and physiologically normal in ovulating women of reproductive age. It is important to consider other causes of a painful adnexal mass (infection, malignancy, ectopic pregnancy, ovarian torsion) in appropriate clinical scenarios. Ruling out occult pregnancy is also important in a young female with adnexal pain. Initial ultrasonography is normal in approximately 15%-26% of women with an ectopic pregnancy. When larger cysts rupture they release blood or cystic fluid that can result in peritoneal irritation and pain. However, expectant management is appropriate because most resolve spontaneously in 8-12 weeks without the need for surgical intervention. Follow-up ultrasonography is recommended in 4-6 weeks. Surgical management is indicated if the cyst persists beyond 8-12 weeks or if other high-risk signs are present on ultrasonography. Combination oral contraceptive pills do inhibit ovulation and prevent cyst formation, but they do not reduce the time it takes for an existing cyst to resolve (SOR B).

A 42-year-old female has experienced progressively worsening pelvic pain for the past 10 years. She experiences dysmenorrhea, dyspareunia, and intermittent pain with bowel movements. A pelvic examination, testing for sexually transmitted infections, and pelvic ultrasonography are negative. Which one of the following is true regarding this problem? A. Behavioral therapy is an integral part of treating chronic pelvic pain B. In patients with chronic pelvic pain, visceral pain is indistinguishable from myofascial pain C. SSRIs may provide pain relief for this patient even if she is not depressed D. Laparoscopy is indicated for all women with chronic pelvic pain

ANSWER: A Physicians should consider the diagnosis of chronic pelvic pain in any woman who has had persistent, noncyclic pain in the pelvic region for longer than 6 months. Studies have reported prevalence rates of 6%-27%, but variation in the definitions used for research on this condition limits assessment of the accuracy of these numbers. The initial evaluation is aimed at identifying underlying conditions that could be causing or contributing to the pain, such as irritable bowel syndrome, endometriosis, interstitial cystitis, or pelvic adhesions. In at least 50% of cases an associated condition can be found. Additional diagnoses to be considered include pelvic floor dysfunction, myofascial pain, degenerative disc disease, chronic constipation, leiomyoma, chronic pelvic inflammatory disease, and nerve entrapment. When an underlying etiology cannot be found, chronic pelvic pain should be treated as a chronic regional pain syndrome. Nearly half of women seeking care for chronic pelvic pain report a history of sexual, physical, or emotional trauma, and about one-third have positive screening results for posttraumatic stress disorder. Asking about these in an empathetic manner is vitally important, as the patient may not offer the history on her own. A careful physical examination should be performed, including a gentle speculum examination with insertion of a cotton swab into the introitus and other labial structures to assess for focal pain and tenderness. The Carnett test can be used to distinguish visceral pain from myofascial pain. For this test the patient tightens the abdominal muscles by raising either both legs or only the head off the table while supine. The examiner places a finger on the painful abdominal site to determine whether the pain increases during the maneuver when the rectus abdominis muscles are contracted. This potentially increases myofascial sources of pain such as trigger points, an entrapped nerve, a hernia, or myositis, whereas visceral sources of pain may be less tender when abdominal muscles are tensed. If severe pain persists, referral for laparoscopy is indicated to diagnose and potentially treat underlying conditions that were not apparent on the initial examination, such as pelvic adhesions or endometriosis (SOR B). It is not indicated for all patients with pelvic pain. Treatment is aimed at improving overall quality of life and function, though evidence supporting specific treatment is limited. What is clear is that a holistic approach is needed, with significant patient engagement. A hybrid approach using cognitive behavioral therapy and physiotherapy, a combination known as somatocognitive therapy, has shown promise. Given the comorbidities of chronic pelvic pain and depression, the evaluation and effective treatment of depression is also an important component of management. Medical management is aimed at symptom relief. Limited evidence supports the use of NSAIDs, tricyclic antidepressants, depot medroxyprogesterone, gabapentin, and SNRIs for the treatment of chronic pelvic pain. SSRIs may provide relief from depression, which may be a comorbidity in patients with chronic pelvic pain, but there is insufficient evidence regarding pain relief. Opioids should rarely be used and only when a comprehensive treatment strategy has failed. While some studies have suggested some benefit from nerve blocks and neuromodulation, the evidence for these is weak.

A 26-year-old female who is 2 months post partum and breastfeeding presents to your office with a history of palpitations, racing heart, and increasing anxiety, which have become progressively worse over the past 2 weeks. A neck examination is normal. Laboratory testing reveals a low TSH level and elevated free T4 and free T3 levels. Which one of the following would be most appropriate? A. A β-blocker B. Levothyroxine (Synthroid) C. Methimazole D. A radioactive iodine uptake scan E. Ultrasonography of the thyroid

ANSWER: A Postpartum thyroiditis is an autoimmune disorder that affects up to 10% of women. It should be considered anytime a woman has symptoms consistent with hyperthyroidism within the first 12 months after delivery, and it can also occur following an abortion or miscarriage. Given that the symptoms can mimic hyperthyroidism caused by Graves disease, the assessment must differentiate which is the underlying cause. While a radioactive I131 uptake study can help distinguish between these two, the use of radiopharmaceutical scanning should be avoided during lactation and radioactive iodine treatment is contraindicated. Antithyroid antibody levels can aid in the diagnosis. There is no need for ultrasonography of the thyroid if the thyroid examination is normal. The hyperthyroid phase of postpartum thyroiditis is caused by autoimmune destruction of the thyroid, causing a release of thyroid hormone. This typically lasts for 1-2 months, followed by a hypothyroid phase lasting 4-6 months. Up to 25% of women will become permanently hypothyroid, whereas the majority will become euthyroid. Antithyroid medications are not helpful, as the hyperthyroid phase of this condition is usually transient. Symptomatic treatment with β-blockers is appropriate for symptomatic patients such as this one. Once in the hypothyroid phase, thyroid hormone replacement is standard. Infants of breastfeeding women should be monitored clinically with respect to their growth and development, but no laboratory monitoring of thyroid function tests is necessary. The American Thyroid Association recommends annual thyroid function testing in women who have had postpartum thyroiditis.

A 35-year-old female presents to your office for treatment of insomnia. You ask if she has experienced any trauma in her life and she discloses that she was sexually assaulted 6 weeks ago. She has not sought medical, legal, or psychological counseling since the assault. During today's visit, you should do which one of the following? A. Assess for symptoms of posttraumatic stress disorder B. Prescribe levonorgestrel (Plan B One-Step), 1.5 mg C. Prescribe HIV postexposure prophylaxis D. Perform a forensic examination to collect evidence, such as a rape kit evaluation E. Refer her for cognitive behavioral therapy

ANSWER: A Sexual assault affects 43.6% of women in the United States during their lifetimes, with increased risks seen in adolescents, college students, LGBTQ persons, and active-duty military personnel. The risk is also increased by physical or mental disabilities, poverty, homelessness, incarceration, and substance use disorders. The majority of assaults are committed by someone known to the victim, and assaults are often unreported. It has been estimated that only 16%-38% of victims seek help from law enforcement or obtain a medical evaluation.Both short- and long-term consequences can occur after sexual assault. Short-term consequences include physical injuries, unintended pregnancy, and sexually transmitted infections (most commonly Chlamydia, gonorrhea, and trichomoniasis). Over time, additional sequelae may include chronic pelvic pain, headaches, fibromyalgia and other chronic pain syndromes, and irritable bowel syndrome. The most common long-term consequence is posttraumatic stress disorder (PTSD), while other psychological sequelae include insomnia, depression, anxiety, substance use disorder, eating disorders, and suicidality. The American College of Obstetrics and Gynecology recommends screening all women for sexual violence, while the U.S. Preventive Services Task Force recommends intimate partner violence (IPV) screening for women of reproductive age. Most women will not disclose IPV or sexual violence unless asked, and a validated two-question screening tool can be most easily incorporated into a primary care practice: "Have you ever been hit, slapped, kicked, or otherwise hurt by your partner? Have you ever been forced to participate in sexual activities?"This patient should have a urine pregnancy test and be tested for Chlamydia, gonorrhea, bacterial vaginosis, syphilis, and trichomoniasis. Blood should be collected for syphilis, HIV, and hepatitis B and C testing. HIV transmission rates depend on prevalence in the community. If this patient had presented within 72 hours after the assault, she would have been eligible for HIV postexposure prophylaxis. If she had presented within 5 days of the assault, provision of emergency contraception would have been appropriate. The diagnosis of PTSD requires at least one re-experiencing symptom, at least one avoidant symptom, at least two arousal and reactivity symptoms, and at least two cognition and mood symptoms. Reported rates of PTSD after sexual assault vary from 30% to 65%. Depression, anxiety, and eating disorders may also be sequelae. Because some psychological symptoms may be long lasting and severe, treating them at the earliest possible time may reduce the duration and severity of distress. Brief screening tools and trauma-specific instruments are frequently used to obtain an objective assessment of specific groups of symptoms such as depression, anxiety, alcohol problems, and posttraumatic stress. The patient should be referred to a counselor with experience in sexual assault (SOR C). Except in situations covered by mandatory reporting laws, patients rather than health care workers make the decision to report a sexual assault to law enforcement. There is no definitive cutoff for obtaining physical evidence after a sexual assault, but it is difficult after more than 48-72 hours even though some DNA evidence is stable for up to 1 week. In addition, most office-based practices do not have an appropriately prepared rape kit or protocols in place to preserve evidence. If the chain of custody is violated the evidence is not admissible for prosecution. Family physicians should be prepared when patients disclose a history of sexual assault and know how to respond in a compassionate and empowering manner. Negative reactions such as blaming the patient, treating the patient differently, and attempting to control the patient's actions can predispose patients to higher rates of depression, substance abuse, and more severe PTSD. A survivor-centered and trauma-informed approach is most effective and includes providing physical and psychological safety, building and maintaining trust with the treatment team, and enabling access to peer support when possible.

You are developing a practice improvement activity in your office centered on substance use disorder (SUD). As part of the training for your clinical staff, you plan to review a variety of clinical vignettes of patients with SUD. One of your goals is to illustrate how SUD has different clinical presentations in women and men. Which one of the following statements is accurate regarding these differences? A. Compared to men, women have a quicker progression from first using a substance to developing dependence B. Compared to men, women with SUD have less severe adverse consequences C. Smaller quantities of drug consumption are associated with development of SUD among men compared to women D. Women are less likely to relapse after treatment than men

ANSWER: A Substance use disorder (SUD) in women is often associated with more severe adverse medical, psychiatric, and functional consequences than in men, often related to the interacting contributions of biological and environmental factors. Physiologically, women with SUD have variation in cravings and drug consumption at different times of the menstrual cycle. There is also evidence that women metabolize nicotine more rapidly than men, making it harder for them to quit using nicotine-containing products. This differential metabolism is a possible reason that nicotine replacement therapies are less efficacious in women. Environmentally, women often attribute their substance use to different reasons than men, including self-treatment of mental health problems, management of chronic pain, and controlling weight. Use of smaller quantities of drugs and a shorter time progression from initial use to dependence are both more likely among women with SUD. Treatment outcomes are not substantially different by sex, but women are more likely to relapse after treatment.

A 24-year-old primiparous woman whose last menstrual period was approximately 8 weeks ago has just screened positive for HIV. Which one of the following statements is true regarding management of HIV infection during pregnancy? A. Pregnant patients who are HIV positive and have a negative hepatitis B surface antigen test should receive the hepatitis B vaccine series B. Because of teratogenicity concerns, a single antiretroviral agent is preferable to a triple antiretroviral regimen C. When the plasma viral load is below the level of detection prior to and throughout the pregnancy, transmission to the fetus does not occur RNA quantitative viral loads should be checked once in each trimester D. Neonatal prophylaxis with zidovudine (Retrovir) should be started within 12 hours of birth and continued for 6 weeks if the mother had an HIV RNA viral load >1,000 copies/mL at the time of delivery

ANSWER: A The CDC recommends universal opt-out prenatal screening for HIV. Repeat testing in the third trimester, before 36 weeks gestation, is recommended for women at high risk for HIV infection, including those living in areas with a high incidence of HIV, those with risk factors for transmission, and those with signs or symptoms of acute HIV infection. A rapid HIV test can be used for women with an unknown HIV status who present in labor. An additional recommendation for pregnant women with HIV is to administer the hepatitis B vaccine series during pregnancy if their hepatitis B surface antigen test is negative on initial screening, and to screen for immunity to hepatitis A and immunize as needed. In addition, those with HIV infection should be tested again for hepatitis B antigen at the time of delivery so that the newborn can be given appropriate therapy if indicated. Inactivated influenza vaccine should be administered during the pregnancy and Tdap should be given at 28-36 weeks gestation. Women diagnosed as HIV positive during pregnancy should be informed that the risk of mother-to-child HIV transmission is estimated at 25%-30%, but that interventions such as antiretroviral therapy, cesarean section, and avoidance of breastfeeding can reduce this risk to less than 1%. The goal for maternal treatment is to reduce the viral load as much as possible, preferably to an undetectable level, as this is the most predictive factor for HIV transmission. Use of combination antiretroviral therapy (cART) during pregnancy in HIV-infected women has been shown to prevent perinatal transmission to the fetus by decreasing maternal viral load and by providing pre-exposure prophylaxis to the infant through placental transfer. The risk of adverse events from cART drugs is small. HIV RNA viral loads should be measured monthly, but testing can be stretched to every 2 months in women whose viral load has remained below the level of detection while on cART. The risk for vertical transmission is lower when the plasma viral load is below detection, but neonatal transmission can still occur. In addition to intrapartum maternal administration of zidovudine, antiretroviral therapy should be given to any neonate of an HIV-infected mother for the first 4-6 weeks of life (SOR A). Four weeks has been shown to be sufficient if the mother's viral load is undetectable, and 6 weeks is recommended if the viral load is measurable or her HIV status is first determined at the time of labor and delivery.

At her 6-week postpartum visit a 25-year-old gravida 2 para 2 reports that she plans to exclusively breastfeed her daughter until 6 months of age. The patient and her husband plan on becoming sexually active again soon, and she asks for advice regarding contraception. Of the following contraceptive choices for this patient, which one has the potential to reduce breast milk production? A. Combination oral contraceptives B. Progestin-only oral contraceptives C. Injectable medroxyprogesterone acetate (Depo-Provera) D. A subdermal progestin implant (Nexplanon) E. A copper IUD (Paragard)

ANSWER: A When reviewing clinical trials and studies regarding the impact that hormonal contraception has on lactation, a Cochrane review concluded that there is limited information on any specific hormonal agent or method and lactation. However, in studies of combined oral contraceptives and lactation, the risk was generally considered to outweigh the benefit and thus may be best avoided. Guidelines from the American College of Obstetricians and Gynecologists acknowledge these concerns but note that the use of combination oral contraceptives may be considered once milk flow is established. Studies of progestin-only methods of contraception (pills, injectable, subdermal, and implant) have generally demonstrated benefits that outweigh the risks with respect to milk production. The CDC supports the opinion that progestin-only contraceptives can be used without concern after 6 weeks post partum. Copper-bearing and levonorgestrel-releasing IUDs can be inserted anytime, including just after delivery, regardless of whether the patient is breastfeeding. There is a higher expulsion rate when placed immediately after delivery and a slightly higher chance of perforation with placement within 6-8 weeks post partum.

A 24-year-old female who just learned that she is pregnant has an ultrasound examination that reveals a delayed early pregnancy loss (missed spontaneous abortion) at 8 weeks gestation. Which one of the following would be the most appropriate advice regarding her options? A. Expectant management is appropriate up to 2 weeks after the diagnosis B. Misoprostol (Cytotec) or uterine aspiration is more effective than expectant management for anembryonic gestation or embryonic demise C. Misoprostol for the treatment of missed spontaneous abortion is effective only when administered vaginally D. Methotrexate is an appropriate medication for missed spontaneous abortion

ANSWER: B A delayed early pregnancy loss (missed spontaneous abortion) refers to demise of an embryo or fetus prior to the 20th week of gestation in which the products of conception remain in utero, typically with a closed internal cervical os. As in this case, ultrasonography will reveal an intrauterine gestational sac (with or without an embryonic/fetal pole), but no embryonic/fetal cardiac activity. The size of the fetus on ultrasonography, not gestational age by last menstrual period dating, is critical information for determining the best management option. Management may be expectant or can be accomplished by a medical or surgical intervention to complete the process. Surgical methods are most effective in managing a miscarriage, followed by medical methods, and then expectant management. Expectant management or placebo has been shown to have the highest chance of serious complications, including the need for unplanned or emergency surgery. It is important to provide medically stable patients with thorough counseling regarding the various management approaches and allow time for the patient to consider the options before making a decision. Some patients have a strong preference for one type of management even before receiving detailed information about the risks and benefits of each.The most effective nonsurgical protocol for managing a delayed early pregnancy loss includes 200 mg of oral mifepristone followed by 800 µg of misoprostol, either vaginally or through buccal administration. Mifepristone is a progesterone receptor antagonist that causes uterine contractions and necrosis of the decidua. Misoprostol is a prostaglandin E1 analogue that ripens the cervix and also causes uterine contractions. While misoprostol can be given alone, the combination is more effective and should be recommended (SOR A). Physical symptoms develop quickly, typically within a few hours of administration, with the most severe occurring within the first 3-5 hours. Light bleeding can continue for up to 9-16 days. There are few contraindications to use of these medications, but patients who have severe cardiovascular, respiratory, hepatic, or renal disease should not be offered this option, and caution should be exercised in considering this combination for a patient with a hemoglobin <10 g/dL. Approximately 8.8% of women who choose medication management with mifepristone and misoprostol will go on to have uterine aspiration due to uncontrolled bleeding or pain. Some women prefer surgical treatment as their initial approach for this reason. Expectant management involves waiting for the products of conception to pass on their own. It is approximately 70%-80% effective in the first trimester and can be continued throughout the first trimester as long as the patient remains clinically stable and shows no signs of infection. A Cochrane review found that rates of infection, as well as mental health outcomes, were similar with expectant and surgical management, but more women required intervention and blood transfusions in the expectant management group. Methotrexate is used in the nonsurgical management of ectopic pregnancy and does not play a role in treating missed spontaneous abortion. A quantitative β-hCG level should be drawn prior to any medical or surgical management so that completion can be confirmed by serial serum β-hCG levels, with the expectation that these levels should fall 50% in the first 24 hours and ≥80% by day 7 after administration.

A 46-year-old female sees you to establish care. Her medical records show that she has a history of cervical intraepithelial neoplasia (CIN) 3 removed by loop electrosurgical excision procedure (LEEP) at age 43 with clear surgical margins. She initially had a Papanicolaou (Pap) smear with HPV co-testing done at 6 months post-LEEP, followed by annual co-testing for the past 2 years, with all results being negative. She is otherwise in good health and takes no medications. Which one of the following should you recommend regarding additional Pap screening? A. Co-testing in 3 years, repeated in 5 years B. Co-testing now, and if normal, repeated every 3 years until 25 years after the initial treatment C. Co-testing now, repeated every 3 years until age 70 D. Colposcopy with endocervical sampling

ANSWER: B After treatment for histologic high-grade squamous intraepithelial lesions (SIL), surveillance should include testing for either HPV alone or co-testing with an HPV test and a Papanicolaou (Pap) smear. Post-treatment HPV testing has been shown to be the most accurate predictor of treatment outcome. Initial post-treatment surveillance should be at 6 months, regardless of the status of the cervical margins, as margin status has been shown to be a poor predictor of persistent or recurrent pre-cancer. After the initial 6-month testing, HPV testing or co-testing should be performed annually until three consecutive negative tests are documented. Beyond this initial surveillance period, continued testing is recommended at 3-year intervals for at least 25 years after treatment of high-grade histology such as this patient's cervical intraepithelial neoplasia (CIN) 3. This prolonged period of testing is recommended even if it goes beyond 65 years of age. The rationale for this screening period is because there is a persistent twofold increased risk for cervical cancer after treatment for high-grade SIL lesions, which persists for ≥25 years and is increased for women over the age of 50.

A 65-year-old female sees you for follow-up to discuss the results of a screening DEXA scan. She is a nonsmoker with no risk factors for osteoporosis. Her height is 165 cm (65 in), her weight is 79 kg (174 lb), and her BMI is 29 kg/m2. Her mean femoral neck density is 0.722 g/cm2, which correlates with a spine T-score of -2.2, and she has a FRAX score of 12% for major osteoporotic fracture and 2.4% for hip fracture. Which one of the following is recommended at this point to reduce this patient's risk of osteoporotic fracture? A. Repeat the DEXA scan in 1 year B. Recommend adequate weight-bearing exercises C. Begin calcium and vitamin D supplementation D. Begin a bisphosphonate such as alendronate (Fosamax) E. Begin raloxifene (Evista)

ANSWER: B All patients should be counseled on risk factors for osteoporosis, including excess alcohol use, a sedentary lifestyle, and secondary osteoporosis related to chronic corticosteroid use, overcorrection of hypothyroidism, or other causes (SOR C). Weight-bearing exercise to prevent falls is a universal recommendation for fracture reduction (SOR C). Recommendations for calcium intake are 1000 mg daily for men 51-70 years of age and 1200 mg daily for women older than 50 years and men older than 70 years (SOR C). However, obtaining these nutrients from dietary sources is preferred, as there is no evidence that calcium or vitamin D supplementation affects fracture rates (SOR B). The National Osteoporosis Foundation recommends starting medication such as a bisphosphonate in women who have a T-score <-2.5 or who have a T-score between -1 and -2.5 and a FRAX score indicating a >3% risk of hip fracture or a >20% risk for total osteoporotic fracture within 10 years (SOR C). This patient does not meet these criteria, but consideration of medication use is reasonable if she has other risk factors for either osteoporotic fracture or falling. Raloxifene, a synthetic estrogen reuptake modulator, has been shown to reduce vertebral fracture rates but not hip fracture rates, and so is not a first-line treatment for the prevention of osteoporotic fracture. It may be considered for women who are unable to take bisphosphonates. It is recommended that DEXA scans not be repeated for at least 2-3 years due to the slow progression of bone remodeling.

A 34-year-old female sees you for the first time. She was diagnosed with type 1 diabetes at the age of 16 and is currently using a combination of isophane insulin (NPH) and regular insulin. She has not seen a doctor for several months, as she has been without medical insurance until recently. She used to take lisinopril (Zestril) for hypertension and atorvastatin (Lipitor) for hyperlipidemia but stopped these medications when her prescriptions lapsed. Her family history is significant for premature coronary artery disease in her mother, who suffered her first myocardial infarction at age 42. On examination the patient's blood pressure is 140/90 mm Hg. Her hemoglobin A1c today is 9.0%. The patient has never been pregnant and mentions that she and her husband would like to have a child in the near future. She asks what she needs to do to maximize her chances of a healthy pregnancy. Which one of the following would be appropriate advice with regard to preparing for pregnancy, in light of her history of type 1 diabetes? A. Resuming lisinopril is important for optimal renal protection B. The optimal hemoglobin A1c goal prior to conception is <6.5% C. Low-dose aspirin should be started now to reduce her risk of preeclampsia during pregnancy D. Diabetic retinopathy stabilizes during pregnancy and evaluation can be postponed until after delivery

ANSWER: B Preconception care is important in all women and imperative in women with preexisting diabetes mellitus. Women must be informed about how diabetes affects pregnancy, and glucose control prior to pregnancy should be optimized through aggressive management. Complications of pregnancy occurring at higher rates in women with either type 1 or type 2 diabetes include preeclampsia, congenital defects, preterm delivery, macrosomia, and stillbirth. Women with type 1 diabetes also have higher rates of diabetic ketoacidosis and cesarean delivery. Establishing good glycemic control prior to pregnancy reduces but does not eliminate these risks. The American Diabetes Association (ADA) recommends a hemoglobin A1c of <6.5% prior to conception for women with diabetes. Women with a hemoglobin A1c >10% should be strongly advised to avoid pregnancy until better control is achieved. Offering monthly hemoglobin A1c levels or continuous glucose monitoring to women who plan to become pregnant soon can help them achieve tighter control of their glucose levels. Assessment for potential teratogenic medications is also critical. Statins, ACE inhibitors, and angiotensin receptor blockers are contraindicated in pregnancy and should be discontinued prior to conception. Patients with hypertension can be switched to an antihypertensive medication that is considered safe for use in pregnancy, such as labetalol, nifedipine, clonidine, or diltiazem. Diltiazem can also decrease proteinuria in patients with preexisting nephropathy. The ADA considers blood pressure goals of 120-135/80-85 mm Hg to be reasonable for avoiding impaired fetal growth. Low-dose aspirin does provide some benefit in preventing preeclampsia in pregnant women at high risk, with a number needed to treat of 19 in a population at highest risk, such as this patient with type 1 diabetes (SOR A). The U.S. Preventive Services Task Force recommends aspirin starting late in the first trimester for women with chronic hypertension, type 1 or 2 diabetes, underlying renal or autoimmune disease, or a multifetal pregnancy. Screening for complications such as retinopathy and nephropathy should be conducted prior to pregnancy. Diabetic retinopathy can worsen during pregnancy, and even though it is likely to improve after pregnancy, any retinopathy progression may worsen vision during the pregnancy. Assessment of a urine albumin/creatinine ratio is also important prior to conception, given that baseline nephropathy is linked to a greater risk of preeclampsia, preterm labor, small-for-gestational age infants, and cesarean delivery.

You recently diagnosed essential hypertension in a 35-year-old female with a strong family history of hypertension. Efforts at lifestyle modification have failed to lower her blood pressure sufficiently, and you determine that antihypertensive therapy is warranted based on serial blood pressure measurements that were all >140/90 mm Hg. She is agreeable to treatment but mentions that she hopes to become pregnant within 6 months. She and her husband have been using a barrier method of contraception. Which one of the following is considered a first-line antihypertensive agent for this patient, given her likelihood of becoming pregnant? A. Atenolol (Tenormin) B. Labetalol (Trandate) C. Lisinopril (Zestril) D. Valsartan (Diovan)

ANSWER: B All women with chronic hypertension should receive preconception counseling that includes recommendations for lifestyle changes used to manage blood pressure, such as regular exercise and dietary changes to achieve a normal BMI. Avoidance of alcohol, tobacco, and illicit drugs, and supplementation with folic acid are also part of this counseling. Given this patient's reproductive plans, it is crucial to choose an antihypertensive medication that is compatible with pregnancy. Labetalol and nifedipine are first-line antihypertensives for managing hypertension during pregnancy, with methyldopa and hydrochlorothiazide deemed safe to consider as second-line medications. β-Blockers and calcium channel blockers appear to be more effective than the alternatives for preventing severe hypertension. However, atenolol should be avoided during pregnancy because of its association with fetal growth restriction. ACE inhibitors and angiotensin receptor blockers are contraindicated in patients who are pregnant or are planning to become pregnant, because these agents are associated with intrauterine growth restriction (IUGR), oligohydramnios, neonatal renal failure, and neonatal death. Women who are on these medications prior to becoming pregnant should be counseled about their potential risks and have their regimen changed to another drug or drug combination. Chronic hypertension is associated with adverse outcomes in pregnancy, including preeclampsia, IUGR, and placental abruption. A 2022 trial demonstrated improved maternal and fetal outcomes (preeclampsia and preterm birth) with treatment of chronic hypertension to a goal blood pressure of 140/90 mm Hg

A 28-year-old female sees you for preconception counseling. She has never been pregnant and takes levothyroxine (Synthroid), 75 µg daily, for hypothyroidism. Her TSH levels have been in the normal range for the past 5 years. Which one of the following is true regarding hypothyroidism in pregnancy? A. TSH levels rise in early pregnancy, peaking at 12-14 weeks gestation B. Monitoring should start as soon as patients have missed a menstrual period or had a positive pregnancy test C. Serum TSH levels should be checked once per trimester for the duration of the pregnancy D. Women with hypothyroidism who become pregnant often need a lower dose of thyroid hormone replacement during the pregnancy E. Adequately treated maternal hypothyroidism is associated with an increased risk for adverse neonatal outcomes

ANSWER: B Among endocrine disorders affecting women of reproductive age, the prevalence of thyroid disease is second only to that of diabetes mellitus. Women with hypothyroidism are advised to achieve euthyroidism before conception because of the risk of lower fertility rates and miscarriage. Untreated or insufficiently treated maternal hypothyroidism is associated with adverse neonatal outcomes, including pregnancy loss, placental abruption, hypertensive disorders, and intrauterine growth restriction. Given that the early gestational period is when the greatest effect of maternal thyroid dysfunction on fetal outcomes occurs, women with pre-existing hypothyroidism should be advised to see their care provider for monitoring as soon as they have missed a period or had a positive pregnancy test. Maternal thyroxine is essential for fetal development, as it supplies thyroid hormone-dependent tissues until maturation of the fetal hypothalamus, pituitary gland, and thyroid gland. This development is nearly complete by 12-14 weeks gestation. After the 14th week fetal brain development may already be irreversibly affected by a lack of thyroid hormones. TSH concentrations are physiologically lower during pregnancy. Beginning at 6 weeks gestation, serum thyroxine-binding globulin (TBG), the major thyroid hormone transport protein in pregnancy, begins to rise due to the estrogen-mediated increase in hepatic TBG synthesis. Maternal thyroid hormone production is also increased by the thyroid stimulating action of placental hCG. TSH levels will usually have begun to fall by the time of the first prenatal visit, and the nadir is reached at 10-12 weeks gestation. For this reason, early and frequent monitoring of free T4 and TSH is warranted, and the levothyroxine dosage should be adjusted as necessary to reach a goal serum TSH level of <2.5 µU/mL (SOR A). This typically involves doubling the daily dose for 2 days each week in early pregnancy (an increase of about 28%). Serum TSH levels should then be measured every 4-6 weeks until 20 weeks gestation and achievement of a stable medication dosage. After this, checking a TSH level at 24-28 weeks and 32-34 weeks is warranted.

A 24-year-old gravida 2 para 1 at 22 weeks gestation presents with a 6-week history of increased wheezing following an upper respiratory infection (URI). She quit smoking before this pregnancy, and she has a history of mild, intermittent asthma since childhood. She has managed her asthma adequately for several years with an albuterol (Proventil, Ventolin) metered-dose inhaler (MDI) that she has needed only about once a month. Since the URI she has been using her albuterol MDI 2-3 times a week. She has continued to have a nonproductive cough but no fever or other symptoms. She says the baby is moving normally. On examination she has wheezing throughout both lung fields without crackles, and has no findings to suggest pharyngitis, otitis media, or sinusitis. Her oxygen saturation is 94% on room air. You assess for proper inhaler technique and persistent allergen exposure and find no concerns. Which one of the following would be the most appropriate management at this point? A. Avoiding additional medications due to potential fetal side effects B. Prescribing low-dose inhaled corticosteroids/formoterol for as-needed use C. Adding theophylline D. Treating her GERD with famotidine (Pepcid)

ANSWER: B Asthma is the most common respiratory condition of pregnancy, with a prevalence rate in the United States of 5%-8%, which is similar to that of nonpregnant women of reproductive age. Asthma control has been shown to worsen for 36% of pregnant women with a history of asthma. Exacerbation rates are linked to underlying severity of the disease: 52% of pregnant women with severe asthma have an exacerbation during pregnancy, compared to 26% with moderate asthma and only 13% with mild asthma. Exacerbations can be triggered by nonadherence to treatment, exposure to allergens or cold, upper respiratory infections, and symptoms that are common during pregnancy, such as rhinitis and gastroesophageal reflux. Additionally, cigarette smoking is associated with a higher incidence of exacerbations. Pregnancy complications associated with asthma include preeclampsia, placental abruption, placenta previa, and obstetric hemorrhage. Higher cesarean delivery rates have also been linked to asthma. This patient has moved from intermittent to persistent asthma. Given her increased as-needed use of a short-acting β-agonist, she should be prescribed low-dose inhaled corticosteroids (ICSs)/formoterol for as-needed use. Evidence has shown this to be more effective at preventing severe exacerbations in patients with mild or infrequent symptoms (e.g., reliever use more than twice a month), such as those seen with viral infections, allergen exposures, poor air quality, or stress (level of evidence A). This is equally efficacious to the addition of daily ICS therapy, which is often fraught with poor adherence in patients with mild asthma. The usual dosage of budesonide/formoterol with mild asthma is a single inhalation of 200/6 µg as needed for symptom relief, with a maximum formoterol dose of 72 µg in a single day. In pregnant women the fetal risk of poor asthma control is much higher than the risks of medication for mild persistent asthma. Additionally, good asthma control during pregnancy has been associated with a lower prevalence of childhood asthma in the offspring. The National Asthma Education and Prevention Program recommends the same management of asthma in pregnant women as in nonpregnant women (SOR B), with long-term goals of achieving good symptom control, maintaining optimal lung function, avoiding maternal hypoxemia, minimizing the risk of acute attacks, and guarding against side effects for the mother and fetus. Theophylline use in the treatment of asthma is quite rare. It may be considered as an alternative agent if a patient's symptoms are not controlled on ICSs and β-agonists and additional step-up therapy is required. However, more women seem to discontinue theophylline use because of its side effects, and it does carry the added burden of monitoring serum concentrations. Studies have consistently shown that it is safer for pregnant women to be treated with asthma medications than to have symptoms and exacerbations. Unfortunately, many women with asthma are not prescribed regular asthma medication during pregnancy. For those who are, it has been estimated that up to 40% do not adhere to the medication regimen while pregnant.

A 22-year-old patient asks you about contraceptive options. She has a history of two elective pregnancy terminations and one spontaneous vaginal delivery. She currently has a hormonal IUD that was placed 1 year ago when she was immediately post partum. She is concerned about the frequency of bleeding she is experiencing. Her medical history is otherwise negative. Which one of the following would be most appropriate for this patient? A. Advising her to keep her IUD in place for a few more months B. Providing balanced information about all contraceptive options in a neutral manner and helping her identify the method that best matches her preferences C. Asking her which contraceptive method she prefers and providing whatever she chooses D. Providing comprehensive written information about all contraceptive options and asking her to follow up with any questions she has

ANSWER: B Contraception is a preference-sensitive decision, and the best method depends on the individual patient's values and preferences. Shared decision-making is a patient-centered approach to counseling that consists of eliciting patients' preferences and then providing them with tailored education and support. Studies have found that patients prefer a shared decision-making approach about contraceptives, and patients who receive contraceptive counseling focused on their preferences are more likely to continue their contraceptive method. Directive counseling in favor of long-acting reversible contraception (LARC) or any other option should be avoided. Studies have documented physician resistance to removal of IUDs, which may be perceived by patients as a violation of reproductive autonomy. Providing patient-centered contraceptive counseling can also promote health equity, as studies have documented higher rates of recommending IUDs to Black and Hispanic women compared to White women. Additionally, Black women are more likely than White women to report that providers pressured them to use contraception. This contributes to distrust of contraceptive providers, with greater than 40% of young Black and Hispanic individuals reporting that they believe that the government promotes birth control to limit minority populations. Providing the patient with written information about all contraceptive options is not the best option in this case, as the patient's health literacy has not been assessed, the physician has not attempted to understand her goals and preferences, and the physician would be relying on the patient's ability to follow up with questions. This would delay addressing her concerns until a later visit, which is not respectful of the patient's time or the additional costs of another appointment.

At a well woman visit, a 46-year-old female mentions that she is no longer interested in having sex with her husband. She reports that her relationship with her husband is good, their communication is excellent, and he is empathetic about her change in sexual interest. She says that she cannot understand why this is occurring, but that these feelings have been consistently present over the last 10 months. She does not have any symptoms of depression or anxiety. Further sexual history indicates that sex is not painful and lubrication is adequate. A physical examination is normal. Laboratory findings, including a metabolic panel and thyroid studies, are normal. You suspect that she has female sexual interest/arousal disorder. Which one of the following has evidence of effectiveness for treating this problem? A. Estrogen cream B. Flibanserin (Addyi) C. Sertraline (Zoloft) D. Sildenafil (Viagra)

ANSWER: B In order for a lack of sexual interest or arousal to qualify as a dysfunction, the problem must be present more than 75% of the time, persist for more than 6 months, and cause significant distress, and must not be explained by other mental health diagnoses, relationship distress, substance abuse, or a medical condition. The DSM-5 criteria for female sexual interest arousal disorder include a lack of, or significantly reduced, sexual interest and arousal, as manifested by at least three of the following: - Absent or reduced interest in sexual activity - Absent or reduced sexual or erotic thoughts or fantasies - Absent or reduced initiation of sexual activity, and typically being unreceptive to a partner's attempts to initiate sexual activity - Absent or reduced sexual excitement or pleasure during sexual activity in almost all sexual encounters - Absent or reduced sexual interest and arousal in response to any internal or external sexual or erotic written, verbal, or visual cues - Absent or reduced genital or nongenital sensations during sexual activity in almost all sex encounters A challenge to diagnosing and treating sexual concerns is that many lack a single etiology. In addition, many cases are complicated by relationship and psychological factors. Dysfunction may be caused by hypothalamic-pituitary axis dysregulation, hypothyroidism, menopause, premature ovarian failure, or musculoskeletal, inflammatory, neurologic, or vascular issues. Sexual function changes can be associated with declining levels of estrogen in peri- and postmenopausal women. Decreases in sexual desire and responsivity have been linked to declining levels of estradiol, while no aspect of sexual functioning has been correlated with measured levels of androgens, including total testosterone, free testosterone index, and dehydroepiandrosterone-sulfate (DHEA-S). The role of testosterone in female sexual dysfunction is not completely understood. Flibanserin has evidence to support its effectiveness for increasing sexual desire in women (SOR B) and was approved in 2015 by the U.S. Food and Drug Administration (FDA) for treatment of generalized acquired hypoactive sexual desire disorder (HSDD) among premenopausal women. Subsequent studies have shown that in naturally postmenopausal women with HSDD, flibanserin has been associated with improvement in sexual desire, improvement in the number of satisfying sexual events, and reduced distress associated with low sexual desire, and it is well tolerated when compared to placebo. However, the FDA has not approved flibanserin for postmenopausal women. Possible side effects include hypotension, dizziness, and syncope. It is not well tolerated with alcohol and is contraindicated in patients who take strong CYP3A4 inhibitors such as fluconazole, verapamil, and erythromycin, or who have liver impairment. Additional treatments being investigated for treating HSDD include testosterone combined with sildenafil or buspirone, bremelanotide, BP101 (a synthetic peptide molecule), and nasal testosterone. Sertraline can interfere with sexual function and would therefore not be appropriate for this patient (SOR C). Estrogen cream is unlikely to be of significant benefit since this patient does not express any concerns with lubrication and there is no evidence of estrogen deficiency on clinical examination.

A 28-year-old nulligravida sees you to discuss infertility. She has no history of sexually transmitted infections. Her male partner has a son from a previous relationship. She has been having unprotected intercourse with her partner for 1 year. Her cycle lengths vary from 28 to 33 days. She would like to know when she is most fertile. Which one of the following would be appropriate advice for this patient regarding ovulation and ovulation monitoring? A. There is strong evidence that continuous monitoring for ovulation increases pregnancy rates B. In each cycle, a woman is most fertile for the 5 days leading up to ovulation C. Basal body temperature and cervical mucus changes are reliable predictors of ovulation D. Urine should be collected in the morning to monitor LH

ANSWER: B Monitoring ovulation and timing intercourse may assist some couples in conceiving but may also lead to significant stress. Most of the data regarding ovulation prediction is unusable or the samples are too small to be of clinical value. Due to survival times of sperm and oocytes, the majority of the fertile window is prior to ovulation, extending from about 5 days before ovulation until several hours after ovulation. Cervical mucus changes occur consistently in some women, but not all. Basal body temperatures can retrospectively confirm ovulation but are not as useful for predicting ovulation in women with irregular cycles. Midday and evening urine samples correlate better with peak LH levels.

A 14-year-old competitive gymnast presents for a well child examination. The patient's mother is concerned that her daughter has not yet started her period. Over the last year the patient has noted breast development and growth of hair in the pubic and axillary areas, and she has had a growth spurt. Which one of the following would be appropriate advice for the patient and her mother? A. The patient should have a laboratory evaluation at this visit B. The patient's pubertal development is within the normal range for girls C. Intense training is not associated with menstrual delay D. Increased skeletal growth typically follows menarche in pubertal development

ANSWER: B Puberty is the process leading to physical and sexual maturation that involves the development of secondary sex characteristics, as well as an increase in height, changes in body composition, and psychosocial maturation. The normal age range for the onset of puberty in girls is 8-14 years. The average age at menarche is 12.5 years, and the absence of any pubertal development by 13 years of age is an indication to evaluate the patient for delayed puberty. This patient has evidence that puberty has begun, including increased skeletal growth, breast development, and the appearance of pubic and axillary hair. Menarche generally follows peak skeletal growth by about a year. Primary amenorrhea is defined as the absence of menarche by age 15, or within 3 years of thelarche. Reassurance would be appropriate for this patient but continued observation is warranted for development of the female athlete triad. This triad is a spectrum of disorders that principally involve three components: low energy availability, menstrual dysfunction, and low bone mineral density.

A 22-year-old female presents with lower abdominal pain and vaginal discharge. She has been sexually active with one male partner for the past 2 months and reports a lifetime total of three partners. She has a levonorgestrel IUD (Mirena) that was placed 2 years ago, and she has no prior history of sexually transmitted infections. She is afebrile, and a pelvic examination demonstrates moderate cervical motion tenderness but no adnexal masses. Which one of the following is true regarding the diagnosis and treatment of this patient? A. Transvaginal ultrasonography should be ordered before treatment B. She should be treated presumptively for pelvic inflammatory disease (PID) without waiting for the results of testing C. Antibiotic regimens that include a fluoroquinolone are recommended for empiric treatment D. The IUD should be removed as soon as the diagnosis of PID is made

ANSWER: B Since 2015, the reported incidence of pelvic infections due to Chlamydia increased by 19% and those due to gonorrhea increased by 53%. The index of suspicion should therefore always be high in sexually active patients with vaginal or pelvic symptoms. This patient is presumed to have pelvic inflammatory disease (PID) based on the history and examination. PID is a clinical diagnosis in women with a supporting history and an examination that demonstrates pelvic pain and either cervical motion tenderness or uterine/adnexal tenderness, as well as the absence of other apparent causes. Uterine or adnexal tenderness, vaginal discharge, fever >38.3°C (100.9°F), and an elevated erythrocyte sedimentation rate and C-reactive protein level also add support for the diagnosis. Ultrasonography is not needed for this patient unless she develops severe symptoms or fails to improve after 72 hours of treatment. Starting empiric antibiotic treatment at the time of presentation is critical to preventing problems such as infertility, chronic pelvic pain, an intra-abdominal abscess, or an ectopic pregnancy. PID can generally be treated in the outpatient setting in women with mild to moderate symptoms (SOR B). Due to increasing macrolide resistance, doxycycline is now recommended for treatment of all chlamydial infections. Similarly, azithromycin is no longer recommended for gonococcal infections, due to a sevenfold increase in resistance. A single intramuscular dose of ceftriaxone for gonococcal infection, along with 14 days of oral doxycycline to treat chlamydial infection is considered the best option for empiric therapy (SOR A). If there is a history of recent uterine instrumentation or either bacterial vaginosis or Trichomonas is found on pelvic examination samples, metronidazole is recommended for 14 days. The emergence of resistance among common pathogens has eliminated the routine use of fluoroquinolones. Expedited partner therapy is recommended, and patients should be advised to abstain from sexual activity until completion of the course of antibiotics and resolution of symptoms. The presence of an IUD does not increase a woman's risk of PID if it is beyond 20 days after the IUD insertion. Treatment outcomes are no worse, and in fact are slightly better, when the IUD is left in place during treatment. Removal should only be considered if there is no clinical improvement in 48-72 hours. Anyone diagnosed with a chlamydial or gonococcal infection should also be tested for HIV and syphilis.

A healthy 32-year-old gravida 2 para 2 sees you because of heavy, painful periods that have increased over the past year. She says she has felt weak and has fatigued easily for the past several months. Her menstrual flow often prevents her from participating in her usual activities. She desires a future pregnancy. A physical examination and orthostatic vital signs are normal. Her hemoglobin is 10 g/dL (N 12-16). Pelvic ultrasonography reveals two large intramural uterine leiomyomas. Which one of the following would be the most appropriate management strategy for her at this time? A. Combination oral contraceptive pills B. A levonorgestrel IUD (Mirena) C. Uterine artery embolization D. Referral for myomectomy

ANSWER: B The incidence of uterine fibroid tumors increases as women age, and they may occur in more than 30% of women 40-60 years of age. Risk factors include nulliparity, obesity, family history, and hypertension. The incidence is higher in older women and those of African descent. Although the pathology of fibroids is benign, they can cause a range of symptoms and signs that need to be addressed, including abnormal uterine bleeding, pelvic pain, constipation, low back pain, urinary frequency, urinary urgency and/or retention, and dyspareunia. Treatment options include medical therapy, uterine artery embolization, myomectomy, and hysterectomy. Treatment must be individualized based on such considerations as the presence and severity of symptoms, the patient's desire for definitive treatment, her desire to preserve childbearing capacity, the importance of uterine preservation, infertility related to uterine cavity distortions, and previous pregnancy complications related to fibroid tumors. This patient is stable and has had no prior treatment. Medical therapy to reduce the volume of menstrual bleeding would be the first choice in this patient, especially in light of her desire for a future pregnancy. Options include hormonal contraceptives, tranexamic acid, and NSAIDs. The levonorgestrel IUD is the most effective medical treatment for reducing tumor volume and blood loss. Within 3 months of starting treatment, 85% of women return to normal bleeding. Heavy bleeding and anemia resolve by 1 year in almost all women. While combination oral contraceptives do significantly reduce blood loss during menses compared to placebo, the levonorgestrel IUD results in a significantly greater reduction in blood loss at 12 months compared to combination oral contraceptives. Hormonal therapy used before surgery has been shown to improve intraoperative and postoperative outcomes by reducing tumor volume and restoring normal hemoglobin.Uterine artery embolization has the advantage of being minimally invasive. However, there is insufficient evidence of its effect on future fertility to recommend it as first-line treatment in women desiring future pregnancy. There is also a higher rate of secondary intervention after uterine artery embolization than after myomectomy. A multicenter, randomized, controlled trial comparing myomectomy to uterine artery embolization for symptomatic fibroids showed that those who underwent myomectomy had a better fibroid-related quality of life at 2 years than those who had uterine artery embolization. Hysterectomy remains the definitive treatment for uterine fibroids and is an appropriate option to consider in women with severe anemia, those who have failed more conservative options, and those who do not wish to have children in the future. When hysterectomy is used to treat uterine fibroids, the least invasive surgical approach should be chosen. GnRH is sometimes used for preoperative management to reduce tumor size and decrease blood loss. Operative and recovery times are reduced when GnRH is used before surgery. Watchful waiting would be appropriate in an asymptomatic patient, or in a woman who is not having vaginal bleeding. Prior to menopause, symptoms will not resolve without treatment

A 29-year-old patient who is 4 weeks post partum and breastfeeds her infant presents with a 1-day history of pain in her left breast, with associated malaise and subjective fever at home. She describes it as feeling like she has "the flu." On examination her temperature is 37.2°C (99.0°F). You note localized redness, swelling, and tenderness of the left breast, but no fluctuance. The right breast is normal for a lactating woman. Which one of the following is true regarding the diagnosis and treatment of this condition? A. Group A β-hemolytic Streptococcus is the most commonly isolated pathogen B. Continued breastfeeding is safe for the newborn during treatment C. A fluoroquinolone would be appropriate antibiotic therapy D. Antibiotics that cover MRSA are recommended

ANSWER: B This patient has mastitis, which is an inflammatory condition of the breast that is usually associated with lactation and commonly occurs in the second and third weeks post partum. Prospective studies have found that this condition occurs in approximately 10% of breastfeeding mothers in the United States. It is concerning not only for the health of the mother, but also for the health of the child in that it can lead to cessation of breastfeeding. Clinical symptoms of mastitis include localized breast pain, redness, and swelling. Fever and malaise may also be present and indicate infection. If an abscess is present, a fluctuant mass may be found. Risk factors include previous mastitis, blocked milk ducts and milk stasis, cracked or sore nipples, yeast infections, a shortened frenulum, and infant attachment difficulties/poor latching. Both prevention and treatment are aimed at optimizing lactation support, including improving breastfeeding techniques and having the mother continue to express milk from the involved breast to avoid further milk stasis and abscess formation. Breastfeeding when mastitis is present poses no risk to the newborn, who is typically already colonized with the same organism at the time mastitis develops. Staphylococcus aureus is the most prevalent organism in mastitis and because few studies are available to specifically guide antibiotic selection, antibiotics that are effective against S. aureus can be selected empirically. Amoxicillin/clavulanate, dicloxacillin, cephalexin, and clindamycin are all effective and safe to use with continued breastfeeding. Clindamycin is the best choice if there is a concern about MRSA; however, this choice should take into consideration local prevalence and sensitivities. Fluoroquinolones are not recommended during lactation.

A 21-year-old primigravida at 12 weeks gestation with no previous history of any drug allergies or urologic problems presented 2 days ago for her first prenatal visit. She reported no urinary tract or systemic symptoms. Today you receive a report that her urine culture is growing >100,000 colonies of a gram-negative rod, with identification and sensitivities to follow. Which one of the following is true regarding this situation? A. Nulliparity is a risk factor for higher rates of asymptomatic bacteriuria in pregnancy B. This condition is associated with an increased risk of low birthweight C. Because the patient is asymptomatic, no further treatment is recommended D. Because the patient is asymptomatic, antibiotic therapy for 3 days with amoxicillin or nitrofurantoin (Macrodantin) is recommended E. A repeat urine culture will be needed for proof of cure

ANSWER: B This patient meets the criteria for asymptomatic bacteriuria (ASB) of pregnancy: >100,000 colonies of a single organism on urine culture with no urinary symptoms. ASB is found in 2%-15% of all pregnancies. Due to the relatively high prevalence and potential complications associated with ASB, the U.S. Preventive Services Task Force and the Infectious Diseases Society of America (IDSA) both recommend that all pregnant women be screened with a urine culture at 12-16 weeks gestation (SOR A). Urine dipstick screening is not recommended because it lacks sensitivity and specificity for this diagnosis. Prospective randomized, controlled trials (RCTs) have shown that antibiotic treatment reduces the incidence of pyelonephritis, recurrence of ASB, and development of symptomatic urinary tract infections (UTIs). Antibiotic treatment is also associated with lower rates of preterm labor and very-low-birthweight infants. Risk factors for ASB in pregnancy include low socioeconomic status, higher parity, diabetes mellitus, a history of recurrent UTIs, and any anatomic abnormalities of the urinary tract. In evaluating the best available evidence for which antibiotic agent is most effective with respect to cure rates, recurrence of infection, and prevention of adverse pregnancy outcomes, a Cochrane review of five RCTs of two antibiotic regimens was unable to draw any definitive conclusions regarding the safety and effectiveness of various treatment options. Nitrofurantoin appears to be the antibiotic choice for ASB during pregnancy, but consideration of cost, local availability, and side effects should help guide selection of the most appropriate antibiotic for each patient. Local resistance patterns of specific pathogens should also be considered, and the antibiotic choice should be based on antimicrobial resistance testing. While the optimal duration of treatment has not been determined, the IDSA recommends a 4- to 7-day course of therapy. A repeat urine culture does not have to be performed if the antibiotic selection was based on culture sensitivities.

A 29-year-old female comes to your office after discovering a mass in her left breast 2 weeks ago. She says it does not hurt and she has no other breast problems. On examination you detect a 3-cm smooth mass in the upper outer quadrant of her left breast. The rest of the examination, including examination of the right breast, is normal. Which one of the following would be the most appropriate next step in the evaluation of this patient? A. Observation only, and follow-up in 4-6 weeks B. Diagnostic mammography C. Breast ultrasonography D. MRI of the breast

ANSWER: C A dominant breast mass is defined as a three-dimensional lesion that is distinct from the surrounding tissues and asymmetric to the opposite breast. A dominant mass increases the index of suspicion for breast malignancy. The goal of the evaluation of a mass is to exclude breast cancer. Physical examination of the breast is not reliable in distinguishing between benign and malignant disease. Up to 10% of women with a breast mass will have cancer and the incidence increases with increasing age and other risk factors. Watchful waiting is not indicated with a dominant breast mass (SOR B) because even though the suspicion of breast cancer is low, it is not zero. In women younger than 30, ultrasonography of the breast is the preferred imaging modality to begin the evaluation (SOR C). For women older than 30, starting with diagnostic mammography is recommended. MRI is rarely indicated in the evaluation of a breast mass (SOR B).

You see a 33-year-old female for a routine evaluation. She tells you that she had a positive home pregnancy test and is hoping to schedule her first prenatal visit. This is her first pregnancy, and although it was unplanned she is very happy. She has been overweight or obese for several years but has not recently been actively dieting or exercising. Her past medical history is otherwise unremarkable. On examination her height is 160 cm (63 in) and she weighs 79 kg (174 lb), with a BMI of 31 kg/m2. Her blood pressure is 110/65 mm Hg and her temperature is 37.0°C (98.6°F). A urine β-hCG in the office is positive. By dates she is at 8 weeks gestation. She says she has not yet gained any weight with this pregnancy and that she would like to try to lose weight at this stage. When discussing the relationship between weight and pregnancy, which one of the following would be appropriate advice for this patient? A. She should not worry too much about weight gain during this pregnancy as long as she eats sensibly and walks daily B. Her maternal risk is not increased, because her BMI is <35 kg/m2 C. The recommended weight gain for her during the pregnancy is 5-9 kg (11-20 lb) D. She is at higher risk for preterm delivery E. Her probability of delivering by cesarean section is the same as for a woman whose weight is classified as normal

ANSWER: C This patient has a BMI of 31 kg/m2, which places her in the obese category. Weight gain during pregnancy should be based on preconception BMI. The National Academy of Medicine recommendation for this patient's target weight gain during the pregnancy is 5-9 kg (11-20 lb). More than 50% of overweight and obese women gain more than this during their pregnancy. Low glycemic and Mediterranean diets are commonly recommended, and a gradual increase in activity to try and reach a goal of 30 minutes of moderate-intensity exercise daily should be encouraged. Exercise has been associated with a reduction in the relative risk for gestational diabetes as well as instrumented and cesarean delivery. Early screening for gestational diabetes should be considered in obese women who are pregnant. This patient's obesity places her at increased risk for miscarriage, thromboembolic disorders, gestational diabetes, a large-for-gestational age (LGA) infant, preeclampsia/eclampsia, and fetal or neonatal death. Her likelihood of operative delivery is increased due to her likelihood of having an LGA infant. Underweight women who conceive with a BMI <19 kg/m2 are at higher risk for preterm delivery, but overweight or obese patients are not.

A 24-year-old nulliparous female who has been using injectable depot medroxyprogesterone acetate (DMPA) contraception for 3 years presents to your office 15 weeks after her last shot. She tells you that she recently changed jobs and that her new work schedule has made it difficult to come in during clinic hours. She is highly satisfied with DMPA and asks if you have any suggestions about how she can continue to use this method. Which one of the following would be most appropriate at this visit? A. Tell her that since she is 2 weeks late for this shot it is not a good method for her, and she should use something else B. Encourage her to switch to an IUD or implant, since those methods are effective and do not require frequent office visits or refills C. Provide her with information about self-injection of DMPA D. Inform her that she should stop using DMPA because of its effects on bone mineral density E. Ask if she has gained weight since starting DMPA, as it is known to cause significant weight gain

ANSWER: C Contraception is a preference-sensitive decision, and physicians should not perform directive counseling towards specific methods based on assumptions about what patients should want. Instead, using a shared decision-making process that focuses on the patient's preferences for contraceptive methods is best for counseling. The goal of patient-centered counseling is to help individuals meet their preferences for when, where, and how they would like to use their chosen method. Since this patient is happy with depot medroxyprogesterone acetate (DMPA) an effort should be made to allow her to continue using it. While the FDA label states that DMPA can be prescribed as a subcutaneous (SC) injection administered by a health care provider, many studies have shown that the safety and efficacy of patient DMPA self-injections are equal to clinic-administered intramuscular (IM) injection. Self-injection has also been associated with higher continuation rates than clinic administration of DMPA. This has led the CDC and World Health Organization to recommend offering DMPA-SC as an additional option for administration. Counseling for this method can be conducted using telehealth or an in-person visit to provide instructions about how to inject, and to observe the first injection if desired. Contraindications and side effects of DMPA-SC are the same as for IM administration and no dose adjustment is needed for either method based on the patient's BMI. There are low rates of adverse events in either formulation, whether user- or provider-administered. According to the CDC's Selected Practice Recommendations for contraception, DMPA injections should be repeated every 3 months or 13 weeks and can be given up to 2 weeks late without requiring a backup method. Both DMPA-SC and DMPA-IM have a window of administration of 12-15 weeks. Studies have demonstrated some decrease in bone mineral density with the use of DMPA. However, bone mineral density increases after discontinuation of DMPA, and cross-sectional and longitudinal studies suggest that it returns to baseline (SOR C). A Cochrane review that examined 22 studies with 11,450 subjects showed limited evidence of changes in weight or body composition with the use of progestin-only contraceptives such as DMPA, compared with another contraceptive method or no contraceptive use. Average weight gain in all of these studies was 2 kg (4 lb) at 6-12 months. Weight gain had doubled at 2-4 years but did not differ significantly among different hormonal treatment groups. Several of these studies were low quality due to a lack of randomization and low continuation rates.

You are seeing a 52-year-old transgender woman for an initial visit. She describes herself as overall healthy and her only medications are spironolactone (Aldactone) and estradiol (Estrace), which she has been taking since age 26. She does not have a family history of cancer. Which one of the following approaches to breast cancer screening for this patient is most consistent with current guidelines? A. No screening for breast cancer B. A clinician-performed breast examination annually C. Mammography every 2 years D. Ultrasonography every 2 years E. MRI every 2 years

ANSWER: C Exposure to estrogen is a risk factor for breast cancer, and transgender women who take feminizing hormones should be considered for breast cancer screening. Current guidelines for breast cancer screening in transgender women are based on retrospective studies, and, similar to guidelines for cisgender women, are not the same across specialty organizations. The Society of Breast Imaging recommends annual mammography starting at age 40 (with no specified duration of hormone exposure), while the UCSF Center of Excellence for Transgender Health recommends mammography every 2 years for women who have been treated with hormones for more than 5-10 years. Of the options listed, the approach that is most consistent with these guidelines is mammography every 2 years. There is no evidence that self-performed or clinician-performed breast examinations are effective screening modalities for transgender or cisgender women. MRI and ultrasonography are not recommended for screening. Physicians should review the different guidelines with each individual and come to a decision together based on patient risk and preference. Patients with a family history of breast cancer should be considered for more frequent screening.

A 52-year-old female sees you because of recurring vulvar itching. She self-treated this problem with over-the-counter antifungal medications multiple times and was subsequently diagnosed with a chronic candidal infection that was treated with several courses of fluconazole (Diflucan). She is not currently sexually active and is not taking any prescription or over-the-counter medications. On a review of systems she notes intermittent pain with defecation. She has no other health problems .On examination you note white atrophic papules and a few plaques on the labia majora. There is some fissuring at the posterior fourchette, making insertion of the speculum painful. The vaginal mucosa appears normal, with a thin, white, normal-appearing discharge. There are whitish plaques and excoriation evident in the perianal region. The remainder of the skin examination is normal. An office wet mount is negative. Which one of the following is the most likely diagnosis? A. Atopic dermatitis B. Atrophic vaginitis C. Lichen sclerosus D. Vitiligo E. Vulvar candidiasis

ANSWER: C Lichen sclerosus (LS) is a chronic, benign, inflammatory dermatologic disorder that most commonly involves the vulvar and anogenital regions. Common symptoms include severe pruritus and dyspareunia, as well as dysuria and a burning sensation or pain with defecation. The etiology is unknown but it most commonly occurs in hypoestrogenic states such as peri- or postmenopause and before puberty. It has also been associated with connective tissue diseases. LS usually manifests as inflammation, epithelial thinning, and dermal changes such as white, thin plaques on the vulvar skin. The disease is progressive in nature, resulting in loss of vulvar architecture and scarring or regression of the labia minora and clitoris if not identified and treated effectively. The initial workup is aimed at ruling out other causes such as infection. Because of its association with squamous cell carcinoma a biopsy is indicated with suspected LS if lesions are persistent or have a more malignant appearance. In addition to relieving symptoms, treatment is also aimed at reversing histologic changes and preventing architectural damage to the vulvar mucosa, so LS should be treated even in women who are asymptomatic. The foundation of therapy is a high-potency corticosteroid ointment (SOR B), most commonly clobetasol 0.05% ointment twice daily until lesions regress, tapering over time. Occasionally, higher-dose corticosteroids such as betamethasone 0.1% ointment are required to achieve symptom relief and resolution. While a single course of therapy can achieve results, multiple or long-term courses may be required. If topical corticosteroids are not effective, a second-line option is topical application of pimecrolimus 1% cream twice daily. Intralesional corticosteroids can also be considered as a later option, and may be effective for more hypertrophic, thickened plaques. Candidiasis is also marked by vulvar pruritus but is usually more erythematous and is less likely to cause pain with defecation. LS is sometimes misdiagnosed as candidiasis, especially if a physical examination is not performed. Atrophic plaques and papules are also less consistent with vulvar candidiasis.Atrophic vaginitis usually presents with vaginal dryness, burning, irritation, and dyspareunia. The tissue appears thin with a loss of rugae, but plaques and papules would be unusual, as would involvement of the perianal region. Vulvar dermatitis is also associated with itching and irritation, but erythema is usually more demarcated and consists of bright red vestibular patches with a hyperkeratotic border. When atopic dermatitis involves the vulva there are usually other signs of dermatitis elsewhere on the skin. Vitiligo causes whitening of the skin but is not characterized by intense pruritus and pain.

A 27-year-old female who is G1P0 sees you because of an episode of painless vaginal bleeding that she describes as heavier than a normal period. She is currently at 8 weeks gestation by reliable menstrual dates but has not yet had an initial prenatal visit. Her vital signs are normal and her uterus is nontender to palpation. On speculum examination no other sources of bleeding are found and only a small amount of blood is present at the cervical os, which is closed. Her β-hCG level is >3000 mIU/mL and ultrasonography confirms an intrauterine 8-week pregnancy with a fetal heart rate of 130 beats/min. No apparent source of bleeding is seen on ultrasonography. Which one of the following is true regarding this patient? A. Her risk for miscarriage is not increased at this stage B. Her risk of preterm labor is not increased C. The presence of cardiac activity in the embryo may be reassuring D. RhoGAM administration is not indicated E. She should remain on bed rest for 1 week beyond the end of the bleeding episode

ANSWER: C Nearly 1 in 4 pregnant women experience vaginal bleeding in the first trimester. Obstetric causes of bleeding include early pregnancy loss and ectopic pregnancy, although bleeding also occurs in viable intrauterine pregnancies. This patient has a threatened abortion with a viable pregnancy and first-trimester bleeding of undetermined etiology. Cardiac activity in the embryo is generally a reassuring sign in patients with first-trimester bleeding, with a probability of miscarriage of only 2.1% in women under age 35, although the probability may be higher in older women. Women who experience bleeding in early pregnancy do have a higher risk of subsequent pregnancy loss overall, with a rate of up to 12%, compared to a risk of 6% of those who do not have bleeding during pregnancy. Additionally, further complications such as preterm labor or intrauterine growth restriction in the same pregnancy occur at slightly higher rates in women who had early pregnancy bleeding. All Rh-negative women should receive 50 µgs of RhoGAM if they experience first trimester bleeding at less than 12 weeks gestation. Bed rest has not been shown to be effective in preventing miscarriage.

A 26-year-old female comes to your office with a 2-year history of infertility and confirmed anovulation. The initial evaluation at today's visit should include which one of the following? A. A karyotype analysis B. A serum progesterone level C. Serum TSH and prolactin levels D. Assessment of the consistency of the cervical mucus E. Pelvic ultrasonography

ANSWER: C Primary ovulatory insufficiency (ovulatory dysfunction) is the most commonly identified cause of infertility in women and is present in up to 40% of cases of infertility. A history and physical examination can identify menstrual disorders, anatomic abnormalities, signs of androgen excess, and other problems that may guide the subsequent evaluation (SOR C). Hypothyroidism and hyperprolactinemia are among the more common causes of anovulation and require early evaluation and specific treatment. Timed FSH and estradiol levels will distinguish between hypothalamic/pituitary, ovarian, and other causes of ovulatory dysfunction. Ovulation should be documented by serum progesterone level measurement at cycle day 21 of a 28-day cycle or 1 week before the predicted onset of menses. Karyotype analysis may be indicated for certain women, such as those with clinical signs of Turner syndrome, but it is not recommended for all women with anovulation (SOR C). While cervical factors may play a minor role in infertility, they are rarely the sole cause. Evaluation of cervical mucus is unreliable and therefore not helpful in the evaluation of infertility. Pelvic ultrasonography can identify follicle development but due to cost and logistics it is not recommended as part of the initial evaluation (SOR C). For patients with a history of endometriosis, pelvic infections, or ectopic pregnancy, evaluation with hysteroscopy or laparoscopy is recommended. The underlying causes of infertility related to endometriosis include anatomic distortions due to adhesions and fibrosis, endocrine abnormalities, and immunologic disturbances. Medical management is not as effective for endometriosis, and surgical treatment or assisted reproductive technology may be required.

You see a 28-year-old gravida 1 para 1 in the office with her 6-month-old for a well child examination. You provided prenatal care for her pregnancy, which was uncomplicated and resulted in a normal vaginal delivery. She recently stopped breastfeeding and asks about the best timing for her next pregnancy. Which one of the following would you recommend as the minimum interval between pregnancies to reduce the risk of complications? A. 9 months B. 12 months C. 18 months D. 24 months E. 36 months

ANSWER: C Short interpregnancy intervals are associated with an increased risk of low birthweight, prematurity, stillbirth, and uterine rupture in patients with a previous history of cesarean section. The results of one meta-analysis indicated that the optimal interpregnancy interval to reduce the likelihood of low birthweight and premature delivery is 18-56 months (SOR A). Some of the potential mechanisms that explain the association between short pregnancy intervals and adverse outcomes include maternal nutritional depletion, folate depletion, cervical insufficiency, vertical transmission of infections, suboptimal lactation related to breastfeeding-pregnancy overlap, incomplete healing of the uterine scar from a previous cesarean delivery, and abnormal remodeling of endometrial blood vessels.

While looking to improve the rates of routine adolescent and young adult immunizations in your practice, you notice your HPV vaccination rates are lower than other primary care practices in your Accountable Care Organization. According to the Advisory Committee on Immunization Practices, your practice should routinely administer HPV vaccine to which one of the following previously unimmunized females? A. A 17-year-old with a history of multiple sexual partners who just found out she is pregnant B. A 27-year-old with atypical squamous cells of undetermined significance on a Papanicolaou smear and a positive test for HPV type 16 C. A 28-year-old with HIV D. A 32-year-old with a new sexual partner

ANSWER: C The ACIP recommends immunizing females against HPV starting at age 11 or 12, but it is possible to start at age 9 (SOR C). The ACIP recommends immunizing males as early as age 11 or 12 and up to age 26 (SOR C). The ACIP also recommends vaccination for everyone through age 26 if not adequately vaccinated previously. HPV vaccine is given as a series of either two or three doses, depending on when the initial dose is given. A two-dose regimen is as effective as three doses if started before age 15. Patients with HIV are higher risk for HPV infections and HPV-related cancers. HPV vaccination thus far appears to be safe in HIV-positive patients, although consistent evidence is lacking for its effectiveness. It is reasonable to consider for any nonimmunized HIV-positive patient, regardless of age. Despite the 32-year-old female's increased risk due to a possible new exposure, HPV vaccine is only approved for routine use up to age 26. While some adults age 27-45 who were not adequately vaccinated when they were younger may request HPV vaccine, this decision should be determined through shared decision-making with their physician. HPV vaccination is less beneficial in this age group, likely because more people in this age range have already been exposed to HPV. HPV vaccines are not licensed for use in adults over the age of 45. HPV vaccination prevents new HPV infections but does not treat existing HPV infections or diseases. However, the vaccine is recommended for women who have had excision of cervical intraepithelial neoplasia (CIN) 2 or higher to reduce recurrent dysplasia. The 27-year-old female should not routinely be vaccinated because of her age, and it will not influence her risks relative to being positive for HPV type 16, but it can be discussed and considered as the vaccine would protect her from infection with other HPV subtypes. HPV vaccine is not approved for use in pregnancy, so it is not recommended for the 17-year-old female.

A 41-year-old female whose pregnancy history is G3P2A1 sees you today to discuss contraception after becoming sexually active with a new partner. You note negative cervical cancer screening 5 years ago. She is currently being treated for hypothyroidism with levothyroxine (Synthroid) and takes no other medications. Recent laboratory studies performed as part of an insurance examination showed a negative HIV test, normal liver function tests, and a blood glucose level of 84 mg/dL. She drinks 4 glasses of wine per week and smokes 1 pack of cigarettes per day. On examination her blood pressure is 122/78 mm Hg and her weight is 59 kg (130 lb), with a BMI of 22 kg/m2. Of the following choices, which one would be the most appropriate contraceptive option for this patient? A. A combined estrogen/progestin pill B. An etonogestrel/ethinyl estradiol vaginal ring (NuvaRing) C. A levonorgestrel IUD (Mirena) D. A norelgestromin/ethinyl estradiol patch (Xulane)

ANSWER: C The CDC U.S. Medical Eligibility Criteria for Contraceptive Use 2016 guidelines use four categories for classifying conditions that are absolute and relative contraindications of contraceptive methods: Category 1: A condition for which there is no restriction for the use of the contraceptive method Category 2: A condition for which the advantages of using the method generally outweigh the theoretical or proven risks Category 3: A condition for which the theoretical or proven risks usually outweigh the advantages of using the method Category 4: A condition that represents an unacceptable health risk if the contraceptive method is used Age ≥35 and smoking 15 or more cigarettes per day is a category 1 condition for the progestin-only pill, the etonogestrel implant, the copper IUD, and the levonorgestrel IUD. A new sexual partner is not a contraindication to IUD use. Age ≥35 and smoking 15 or more cigarettes per day is a category 4 condition for estrogen-containing contraceptives. Patients taking combined (estrogen/progestin) contraceptives who smoke are at increased risk for myocardial infarction compared with those who do not smoke. Studies also showed that the risk for myocardial infarction increases with the number of cigarettes smoked per day. This risk assessment must also take into account whether the patient has other risk factors for myocardial infarction or cardiovascular disease, such as hypertension. Women younger than 35 who smoke can generally use combined oral contraceptives (category 2) but should also be screened for other conditions that may alter their risk category. This patient's hypothyroidism does not have an impact on contraceptive choice, as this is a category 1 condition for all options listed. While she is due for cervical cancer screening, this should not be required before prescribing hormonal contraception (SOR C).

A 22-year-old female sees you to establish care after moving to your area. She reports a recent history of new headaches and describes a unilateral visual disturbance that is scintillating and starts before the headache. This disturbance usually lasts about 30 minutes and resolves before the headache pain begins. She has never been pregnant and does not wish to become pregnant in the next few years. Her medications include norgestimate/ethinyl estradiol, 0.25 mg/35 µg. She was diagnosed with pelvic inflammatory disease 5 months ago and completed treatment. She is a nonsmoker and her vital signs are normal today. According to the CDC's 2016 Medical Eligibility Criteria for Contraception Use, which one of the following contraceptive options would be CONTRAINDICATED for this patient? A. Medroxyprogesterone acetate (Depo-Provera) B. An etonogestrel implant (Nexplanon) C. An etonogestrel/ethinyl estradiol vaginal ring (NuvaRing) D. A copper IUD (Paragard) E. A levonorgestrel IUD (Mirena)

ANSWER: C The etonogestrel/ethinyl estradiol vaginal ring would be contraindicated in this patient, who has migraine with aura. The CDC and the World Health Organization provide guidelines regarding considerations and contraindications for contraception in women, using the following classification system: Category 1: A condition for which there is no restriction for the use of the contraceptive method Category 2: A condition for which the advantages of using the method generally outweigh the theoretical or proven risks Category 3: A condition for which the theoretical or proven risks usually outweigh the advantages of using the method Category 4: A condition that represents an unacceptable health risk if the contraceptive method is used Migraine with aura is a category 4 condition for any estrogen-containing contraception. This patient's current combined hormone pills should be stopped and she should be switched to a non-estrogen-containing contraceptive. It is not appropriate to switch her to a combined pill with a lower dose of estrogen or to initiate the etonogestrel/ethinyl estradiol vaginal ring or the norelgestromin/ethinyl estradiol patch. Migraine with aura is a category 1 condition for initiation of the levonorgestrel IUD, copper IUD, or etonogestrel implant, as well as for initiation of medroxyprogesterone acetate. This patient's nulliparous status is category 2, and her history of pelvic inflammatory disease more than 3 months ago is also category 2.

A 25-year-old female who self-identifies as lesbian comes to your office for a health maintenance examination. She received the recommended doses of HPV vaccine at age 12. An HIV test was negative 18 months ago. Her most recent Papanicolaou smear 1 year ago was negative. She has no medical concerns today. Her blood pressure is 120/72 mm Hg, her BMI is 23 kg/m2, and a physical examination is normal. She intermittently uses dental dams. Appropriate care at this time includes which one of the following? A. HPV testing B. Repeat HIV testing C. Screening for intimate partner violence D. Screening for bacterial vaginosis

ANSWER: C The majority of screening recommendations are the same for women who have sex with women (WSW) as for other women. WSW should be screened for cervical cancer according to the same guidelines as women who have sex with men, beginning at age 21. Studies have shown that WSW have lower rates of cervical cancer screening and tend to begin screening later, with the primary reason being fear of discrimination. This lower rate of screening may increase cervical cancer in this population. Additionally, the majority of self-identified WSW indicate that they have had sex with men at some point in their life, so HPV and Papanicolaou testing can be positive for precancerous lesions. Finally, HPV can be transmitted through skin-to-skin contact and can be sexually transmitted between women, so WSW should not be excluded from HPV testing or co-testing. Guidelines for HPV vaccination in WSW are also the same as in other women. The CDC recommends HPV vaccine for females 11 and 12 years of age (although it can be given as young as 9 years of age), as well as for females 13-26 years of age who have not yet received all doses or completed the vaccine series. WSW should receive the same sexually transmitted infection screenings as bisexual or heterosexual women. Rates of Chlamydia infection are likely higher than previously thought in WSW, and female-to-female transmission of syphilis and gonorrhea does occur. Although the frequency of bacterial vaginosis in WSW populations ranges from 25%-52%, routine screening is not recommended by the CDC. Rates of depression, anxiety, and suicidal ideation and attempts are higher in WSW than in heterosexual women. This is especially true for sexual minority adolescents and in WSW who have been the target of prejudice or discrimination. Studies have also reported higher rates of intimate partner violence among WSW, with a reported lifetime prevalence of 43.8% compared to 35.0% for heterosexual women. Use of screening tools for depression and intimate partner violence in WSW should be comparable to their use in bisexual and heterosexual patients.

A 30-year-old primigravida at 32 weeks gestation sees you because of generalized pruritus for the past week that has become intense enough to interfere with sleep. It is most intense on her palms and soles. There are no visible lesions on her skin, and she does not appear to be jaundiced. She has no other medical problems. Fetal movement, heart rate, and fundal height are appropriate for gestational age. Laboratory findings include the following: Total bilirubin............1.1 mg/dL (N 0.1-1.2) Albumin............3.6 g/dL (N 3.1-4.7) Total protein............6.5 g/dL (N 6.0-8.0) Alkaline phosphatase............200 U/L (N 40-125) ALT............60 U/L (N 10-39) AST............68 U/L (N 4-42) Serum bile acids............21 µg/mL (N <12) Which one of the following is true regarding this situation? A. A characteristic rash aids in making the diagnosis B. She should have a liver biopsy C. Treatment with ursodeoxycholic acid can benefit both mother and fetus D. Cholestyramine (Questran) is first-line therapy E. Early induction of labor does not reduce perinatal morbidity or mortality

ANSWER: C This patient has intrahepatic cholestasis of pregnancy (ICP), evidenced by generalized pruritus, elevated liver enzymes, and increased serum bile salts, generally in the absence of a rash (other than evidence of scratching). This is a pregnancy-specific reversible condition that results from impaired metabolism and excretion of bile acids from the maternal bloodstream. An interplay of genetic, endocrine, and environmental factors appears to play a role in the pathogenesis. Symptoms begin in the second or third trimester of pregnancy and resolve very quickly following delivery, resulting in a generally benign course for the mother. Fetal complications are not benign, and include spontaneous preterm delivery, meconium staining of the amniotic fluid, intrauterine fetal demise, and respiratory distress syndrome. In cases of intrauterine fetal demise, which can occur in up to 15% of untreated cases, autopsy findings are typically consistent with signs of acute, but not chronic, anoxia. The incidence of ICP in the United States is about 7 in 1000 pregnancies. Laboratory parameters include total bile acid levels that are 10-25 times normal levels, along with mild elevations in liver enzymes. Serum bilirubin rarely reaches 6 mg/dL. Pathologic findings are not specific and a liver biopsy is rarely indicated.Stillbirth is not possible to predict, as it may occur within 2 days of reactive nonstress testing and with normal fetal kick counts. Stillbirth usually occurs after 37 weeks gestational age; hence the primary consideration in management is delivery no later than 38 weeks gestation, and earlier if the case is severe and fetal lung maturity has been demonstrated (SOR C). Careful monitoring of the progression of the disease and pregnancy is necessary and includes maternal liver function tests, serum bile acid levels, and ongoing assessment of fetal well-being. Several medications have been used to treat maternal symptoms during pregnancy. Ursodeoxycholic acid, 500 mg twice daily, has been shown to provide symptomatic relief from pruritus, decrease elevated liver enzymes and maternal serum bile salts (SOR B), and improve fetal outcomes. Cholestyramine, once considered a mainstay of treatment, provides symptomatic relief for the mother, but does not lower liver functions or bile salts and has no effect on fetal outcome.

A 67-year-old female sees you because of urinary incontinence. She reports an 8-month history of intermittent episodes of an abrupt need to void, accompanied by leakage of urine before she has a chance to get to the bathroom. She has not had any leakage of urine with coughing, sneezing, or laughing. She has mild hypertension controlled with amlodipine (Norvasc), 5 mg daily. Otherwise, she has no medical problems and takes no other medications. An examination is normal except for vaginal dryness. A urinalysis is negative for WBCs, nitrites, and bacteria. Which one of the following interventions would be most appropriate for this patient? A. α-Adrenergic agonists such as pseudoephedrine or phenylephrine B. β-Adrenergic agonists such as mirabegron (Myrbetriq) C. Pelvic floor muscle exercises D. Posterior tibial nerve stimulators E. Retropubic urethropexy

ANSWER: C Urinary incontinence is a common problem in the primary care setting, as it occurs in some form in approximately half of the women in the United States. The initial evaluation is aimed at identifying the length of time the patient has experienced incontinence, the type of incontinence, and any red flag findings that would warrant a more immediate in-depth workup. This patient's history is consistent with urge incontinence, which involves an abrupt desire to void accompanied by an involuntary leakage of urine. There are no red flag signs on examination and the urinalysis does not suggest an acute or chronic infection. An assessment of current medications and other substances that can cause or exacerbate incontinence symptoms should be sought. For urge incontinence, this includes caffeine, alcohol, and diuretics. Calcium channel blockers can decrease bladder contractility and may contribute to urinary retention and overflow incontinence, but not urge incontinence. Conservative management should be the first-line approach in women with either urge or stress incontinence (SOR C). Pharmacologic agents should only be used as an adjunct to behavioral approaches (SOR C). Patients should also be advised about fluid intake, choosing noncaffeinated and noncarbonated beverages, timed voiding, the benefit of regular exercise as well as weight loss in women who are obese or overweight, and smoking cessation. Pelvic floor muscle strengthening with Kegel exercises and/or pelvic muscle tone training should be tried for a suitable time before considering medication, unless there are red flag signs present. Red flag signs include pain, hematuria or proteinuria, pelvic organ prolapse, a history of previous pelvic surgery or radiation, suspected fistula, and an elevated postvoid residual volume. Properly performed pelvic floor muscle exercises are more effective than medications for reducing urge incontinence (SOR B). If conservative treatment fails, β-adrenergic agonists such as mirabegron can be used to treat urge incontinence. It acts on β1-adrenergic receptors to relax the detrusor. Older patients may prefer mirabegron over the potential side effects of anticholinergic drugs. Mirabegron can increase blood pressure, however, and should not be used in patients with uncontrolled hypertension. Other adverse effects of mirabegron include nausea, diarrhea, constipation, dizziness, and headache. Since this patient has not had a trial of conservative therapy, this would not be the most appropriate choice after her initial evaluation. The FDA has approved several electrical neuromodulation devices for treating urge incontinence that does not respond to behavioral interventions. Percutaneous stimulators of the posterior tibial nerve, which shares a common nerve root with innervation of the bladder, are the most widely used devices. Posterior tibial nerve stimulation works at least as well as medication, reducing urge incontinence in up to 75% of patients (SOR B). However, this requires weekly procedures for the first 3 months and monthly treatments thereafter. α-Adrenergic agonists, such as pseudoephedrine and phenylephrine, cause urethral constriction but are not indicated for the treatment of urge incontinence because there is only weak evidence to support their superiority over placebo (SOR B). Urethropexy, a surgical procedure to provide support to the urethra, is used for treatment of stress incontinence that does not respond to less invasive treatments (SOR B), but it is not used to treat urge incontinence.

An 18-year-old female sees you for a prenatal visit at 38 weeks gestation. She asks about postpartum contraceptive options and mentions that she values ease of use and effectiveness. She is not interested in having an IUD placed. Her BMI is 34 kg/m2. She has no medical problems and shares that she is not planning to breastfeed because her schedule is unpredictable. Which one of the following contraceptive options would be safest and most effective for this patient? A. A progestin-only pill B. A combined estrogen/progestin pill C. An etonogestrel/ethinyl estradiol vaginal ring (NuvaRing) D. An etonogestrel implant (Nexplanon) E. A norelgestromin/ethinyl estradiol patch (Xulane)

ANSWER: D According to the U.S. Medical Eligibility Criteria for Contraceptive Use 2016, contraceptive implants, medroxyprogesterone acetate, progesterone-only pills, and IUDs (either levonorgestrel or copper) are suitable methods with advantages that outweigh the theoretical or proven risks for women who are less than 21 days post partum and not breastfeeding. Most contraceptive types and methods can be used soon after pregnancy with the exception of estrogen-containing contraceptives, which should be delayed until after 3 weeks post partum due to the increased risk of venous thromboembolic disease. Postpartum initiation of long-acting reversible contraceptive methods (LARCs) is highly effective at preventing short interdelivery intervals. Oral, patch, or ring methods are associated with rates of interdelivery intervals that are similar to users of no prescription contraception. LARCs are user-independent and can be safely placed immediately or soon after delivery, even in adolescents, with no increased risk of complications. LARCs are more effective than any other method and are comparable to permanent contraception. Since this patient does not wish to have an IUD placed, the etonogestrel implant would be the most appropriate choice. Progesterone-only oral contraceptives have often been utilized in the postpartum period in mothers who are breastfeeding, as they were felt to have less impact on breastmilk production. While they continue to be a safe option, they have a 9% failure rate, making them less effective than LARCs. In women with a BMI >30 kg/m2, there is no restriction on the use of IUDs, implants, medroxyprogesterone acetate, or progestin-only contraceptives. There was previously some concern that some combined oral contraceptive methods were less effective with increasing BMI, but the data is conflicting. A 2016 Cochrane analysis of 17 studies that included more than 68,000 women found no association of BMI or weight with the effectiveness of hormonal methods of contraception. Traditionally, the discussion about postpartum contraception occurs at the 6-week postpartum visit. However, in women who are not exclusively breastfeeding, ovulation resumes as early as 21 days after childbirth. For this reason, it is important to have a plan for contraception established before delivery so that the preferred method, determined through shared decision-making, can be offered immediately postpartum. Failure to do so puts women at high risk of unplanned pregnancy, with the concomitant risks associated with short interpregnancy intervals, such as small-for-gestational age infants, preterm birth, and infant mortality.

A 29-year-old female comes to your office with symptoms of an upper respiratory infection (URI). During the visit she mentions that she ran out of birth control pills 2 months ago and she wants to restart contraception. The last time she had intercourse was 4 days ago and her male partner did not use condoms. Her last menstrual period was 2 weeks ago. Her past medical history is negative for any chronic conditions. Her vital signs are normal and a physical examination is otherwise unremarkable except for the URI. A urine pregnancy test is negative. After reviewing various options for contraception, she asks which one would be most effective in preventing pregnancy. You should recommend A. a combined estrogen/progestin pill B. a norelgestromin/ethinyl estradiol patch (Xulane) C. an etonogestrel/ethinyl estradiol vaginal ring (NuvaRing) D. a copper IUD (Paragard)

ANSWER: D Any of the above options could be used in this situation, and the best choice for this patient depends on her preference. Her reasons for previously discontinuing birth control pills may help guide shared decision-making. In terms of overall effectiveness, insertion of a copper IUD is the most effective form of birth control and can serve the dual purpose of emergency contraception in patients who have had unprotected intercourse within 7 days of the office visit. The copper IUD would therefore be preferred in this case. New evidence suggests that the levonorgestrel IUD may also be used in this situation. However, any type of hormonal contraception can be started safely and effectively at any point in the menstrual cycle, after obtaining a negative pregnancy test. This increases adherence, improves initiation time, and reduces unwanted pregnancies. Patients should be advised that while an early pregnancy is possible, it would not be harmed by the use of hormonal contraception. The patient should use a back-up method for 7 days and have a repeat pregnancy test in 2-4 weeks. Continuation rates are enhanced when patients take their first pill immediately after requesting birth control (SOR C). If the unprotected sex occurred within 5 days of the visit, ulipristal can be prescribed as emergency contraception, and hormonal contraception can be started 5 days after the patient takes the ulipristal.

A 22-year-old female comes to your office following an unprotected sexual encounter 2 nights ago. She does not wish to become pregnant. Safe and effective options for emergency contraception include all of the following EXCEPT A. levonorgestrel (Plan B One-Step) B. ulipristal (Ella) C. combined oral contraceptive pills (Yuzpe method) D. medroxyprogesterone injection (Depo-Provera) E. the copper IUD (Paragard)

ANSWER: D Emergency contraception is used to prevent pregnancy after an unprotected or inadequately protected act of sexual intercourse. Oral mifepristone, ulipristal, levonorgestrel-releasing emergency contraception, ethinyl estradiol/levonorgestrel, and both the copper and levonorgestrel IUDs are safe and effective for emergency contraception. Medroxyprogesterone injection has not been shown to be effective in the management of emergency contraception. The most commonly used oral emergency contraceptive regimen is the progestin-only pill, containing levonorgestrel, which is available over the counter with no age restriction. Ulipristal is a selective progesterone receptor modulator that is approved by the FDA for emergency contraception for up to 120 hours after coitus and requires a prescription. It is slightly more effective than levonorgestrel for preventing unintended pregnancies if prescribed within 72 hours (SOR A). Both levonorgestrel and, to a lesser degree, ulipristal are less effective in women with a higher BMI. The Yuzpe method may also be used, consisting of a combined oral contraceptive containing 100 µg ethinyl estradiol and 0.5 mg levonorgestrel (equivalent to 1 mg norgestrel) given as two doses 12 hours apart within 72 hours of intercourse. This can be advantageous for patients who already have pills at home. The number needed to treat to prevent one pregnancy is 17. Oral mifepristone (25-50 mg) is also a more effective method of emergency contraception than other oral forms. The copper IUD is the most effective method used for emergency contraception (0.09% failure rate) and it should be considered in women who are not at high risk for sexually transmitted infections (SOR A). An added advantage is that it can be placed within 7 days of sexual intercourse (SOR A). It also provides continued contraception and is not affected by the patient's weight.

A 28-year-old patient who recently immigrated from Honduras comes to your clinic in November to establish care. She is unaware of whether or not she received any childhood immunizations and has no way of obtaining her childhood health records. She does not recall ever having had varicella. She has no known allergies or medical problems and she does not use tobacco. She was recently hired as a patient-care aide in a nursing home and will begin that job in 2 weeks. She has recently married, is using oral contraceptives, and states that she hopes to become pregnant within the next 6-12 months. Her last menstrual period was normal and ended 3 days ago. Which one of the following would NOT be indicated for this patient at this time? A. COVID-19 vaccine B. Inactivated influenza vaccine C. MMR D. Pneumococcal vaccine E. Tdap

ANSWER: D Given the lack of documentation and uncertainty about this patient's medical history, she should be treated as if she has had no prior vaccines. Vaccination prior to pregnancy will help prevent infections in the mother, and any vaccines administered during pregnancy may help protect the fetus because of placental transfer of maternal antibodies and will continue to offer protection with breastfeeding. Due to the increased risk of maternal and fetal morbidity and mortality, the inactivated influenza vaccine is recommended for all women who may or will be pregnant during influenza season and do not have contraindications to the vaccine. Similarly, being vaccinated against SARS-CoV-2 prior to pregnancy should confer immunity against COVID-19 and prevent potential morbidity and mortality in the patient and her offspring. In the vast majority of adults with no clear history of varicella, a titer will show that they are immune, so it is cost-effective to draw the titer initially and vaccinate only if it is negative. For nonpregnant women with no evidence of immunity, one dose of MMR is recommended. This vaccine is contraindicated during pregnancy, so it would be best to administer it now. This patient should be cautioned not to become pregnant within 4 weeks of receiving MMR, because it contains attenuated live virus and is therefore potentially teratogenic. For patients who did not receive the primary series for tetanus, diphtheria, or pertussis, at least one dose of Tdap should be given now, followed by one dose of Td or Tdap at least 4 weeks later, and a third dose of Td or Tdap 6-12 months after the second dose. While Tdap can be substituted for any Td dose, it is preferred as the first dose. Following this series, she should receive Td or Tdap every 10 years. Tdap is also recommended for all pregnant women between 27 and 36 weeks gestation. Pneumococcal vaccination is indicated only for patients age 65 and older who are immunocompetent or those between the ages of 19-64 with an underlying immunocompromising condition or a chronic medical condition such as diabetes mellitus; heart, lung, or liver disease; substance use disorder; or cigarette smoking. This patient appears to have none of these indications and should not be considered for pneumococcal vaccine at this point.

A 53-year-old female sees you for treatment of menopausal hot flashes. She is not interested in hormonal treatment because she has a sister with breast cancer. Which one of the following nonhormonal methods has the best evidence for relieving this patient's vasomotor symptoms? A. Exercise B. Acupuncture C. Black cohosh D. Paroxetine (Paxil) E. Cognitive behavioral therapy

ANSWER: D SSRIs are the most efficacious nonhormonal medication option for managing hot flashes. Venlafaxine, desvenlafaxine, paroxetine, citalopram, and escitalopram have a similar modest benefit for hot flashes, but low-dose paroxetine, 7.5 mg daily, is the only agent approved by the FDA for the treatment of hot flashes. Paroxetine should not be used in women taking tamoxifen, as it is a potent inhibitor of CYP2D6, the enzyme that converts tamoxifen to its most active metabolite. Other nonhormonal medications that have been effective for relieving hot flashes include gabapentin and pregabalin, but the associated dizziness and other adverse effects limit their use at effective dosages. Cognitive behavioral therapy can improve the severity of vasomotor symptoms but does not reduce their frequency. Evidence is insufficient to show the effectiveness of relaxation techniques as treatment for menopausal vasomotor symptoms, or to determine whether this treatment is more effective than no treatment or placebo. Randomized trials have not shown any benefit from regular exercise in relieving vasomotor symptoms. There is no high-quality evidence that demonstrates that black cohosh or other botanical agents, omega-3 fatty acids, or lifestyle modification consistently provide benefit in alleviating hot flashes (SOR B). Acupuncture, when compared with sham acupuncture, showed no evidence of greater effectiveness for alleviation of hot flashes.

A 62-year-old female with well controlled hypertension comes to your office for follow-up. She has no history of cardiovascular disease. Her medications include lisinopril/hydrochlorothiazide (Zestoretic) and acetaminophen. She has smoked half a pack of cigarettes daily for the last 45 years. She has a BMI of 38 kg/m2 and her blood pressure is 134/82 mm Hg. A physical examination is normal, including a normal rate and rhythm with no murmurs on cardiac examination. Results of a lipid panel include an LDL-cholesterol level of 220 mg/dL, an HDL-cholesterol level of 36 mg/dL, and a triglyceride level of 140 mg/dL. Her calculated 10-year risk of cardiovascular disease (CVD) is 21.3%. You counsel the patient on the importance of smoking cessation. Which one of the following pharmacologic options for CVD prevention recommended by the American Heart Association would be most appropriate for this patient? A. Apixaban (Eliquis) B. Low-dose aspirin C. Clopidogrel (Plavix) D. Atorvastatin (Lipitor)

ANSWER: D Smoking cessation is a critical component of decreasing the risk for cardiovascular disease and stroke. In addition, the American Heart Association and the American College of Cardiology recommend statin therapy as first-line management for the primary prevention of atherosclerotic cardiovascular disease (ASCVD) in patients with LDL-cholesterol levels >190 mg/dL, those with diabetes mellitus, and those who have a high calculated ASCVD risk. This patient's 10-year risk of heart disease is elevated, with risk factors including hypertension, age, obesity, and smoking, placing her in a high-risk category. Aspirin should not be started for routine primary prevention of ASCVD because of a lack of demonstrated benefit. The U.S. Preventive Services Task Force has made the use of low-dose aspirin for primary ASCVD prevention in adults over 60 years of age a D recommendation, which updates their 2016 recommendation for shared decision-making for patients age 60-69. Antiplatelet agents such as clopidogrel do not play a role in primary prevention and should be reserved for secondary stroke prevention or for use in those allergic to aspirin. The use of apixaban would be reasonable to consider if this patient had atrial fibrillation.

You are helping your practice develop a quality and safety monitoring program for prescribing contraception. In the CDC U.S. Medical Eligibility Criteria for Contraceptive Use 2016 guidelines, category 4 indicates conditions for which a particular contraceptive method has an unacceptable risk to the user. Which one of the following conditions is classified as category 4 for combined hormonal contraception? A. Controlled hypertension B. Diabetes mellitus with end-organ damage C. Systemic lupus erythematosus with positive antiphospholipid antibodies D. The use of lamotrigine (Lamictal) in patients with a stable seizure disorder E. A family history of breast cancer

ANSWER: D The CDC U.S. Medical Eligibility Criteria for Contraceptive Use 2016 guidelines use four categories for classifying conditions that are absolute and relative contraindications of contraceptive methods: Category 1: A condition for which there is no restriction for the use of the contraceptive method Category 2: A condition for which the advantages of using the method generally outweigh the theoretical or proven risks Category 3: A condition for which the theoretical or proven risks usually outweigh the advantages of using the method Category 4: A condition that represents an unacceptable health risk if the contraceptive method is used A family history of breast cancer is a category 1 condition for the use of combined hormonal contraceptives. Adequately controlled hypertension, diabetes mellitus with end-organ damage or a duration >20 years, and the use of lamotrigine with a stable seizure disorder are all category 3 conditions, and systemic lupus erythematosus with positive antiphospholipid antibodies is a category 4 condition.

A 47-year-old gravida 2 para 2 reports a new onset of spotting between periods for the last 6 months. She has moderate osteoarthritis of her knee from a past injury but has no other health problems. Her only medication is ibuprofen, 400 mg twice daily, and she has had a tubal ligation. She has no history of postpartum hemorrhage, bleeding related to surgery, or easy bruising. She has a BMI of 26 kg/m2 and an examination, including a pelvic examination, is normal. A urine pregnancy test is negative. Her TSH level, prolactin level, and hematocrit are normal. Which one of the following would be most appropriate at this point? A. Reassurance that this is normal menopause B. Placement of a copper IUD C. Transvaginal ultrasonography D. An endometrial biopsy E. MRI

ANSWER: D The PALM-COEIN classification system can be used to categorize the most common causes of abnormal uterine bleeding. PALM refers to structural causes: P-endometrial polyps, A-adenomyosis, L-leiomyoma, and M-malignancy or hyperplasia. COEIN refers to nonstructural causes: C-coagulopathy, O-ovulatory dysfunction, E-endometrial, I-iatrogenic, and N-not yet classified. After initial evaluation with a detailed history, pelvic examination, pregnancy test, and CBC, an endometrial biopsy is indicated in women ≥45 years of age, or in younger women with risk factors for endometrial cancer or significant unopposed estrogen exposure. Other risk factors include obesity, nulliparity, polycystic ovary syndrome, type 2 diabetes, hypertension, thyroid disease, or cholelithiasis. This patient should have a biopsy for tissue sampling rather than transvaginal ultrasonography because she is at risk for endometrial cancer and intermenstrual bleeding is not a normal part of perimenopause. MRI is not a primary imaging modality for abnormal uterine bleeding. Medical management of abnormal uterine bleeding is appropriate in women <45 years of age with no risk factors for hyperplasia or uterine cancer, and/or the absence of prolonged anovulation associated with unopposed estrogen. Use of a levonorgestrel-releasing IUD is most effective for decreasing heavy menstrual bleeding (71%-95% reduction in blood loss) and performs similarly to hysterectomy when quality-adjusted life years are considered. Other options include combined estrogen-progesterone contraceptive pills, medroxyprogesterone, NSAIDs, oral progestins, and oral tranexamic acid.

A 28-year-old nulligravida has been unable to conceive after trying for 15 months. You review her menstrual history and medical history, and as part of an infertility workup you recommend a progesterone level 1 week before her expected menses to confirm that she is ovulating. Which one of the following phases of the menstrual cycle is characterized by high progesterone levels and low FSH and LH levels? A. The follicular-proliferative phase B. The follicular-secretory phase C. The luteal-proliferative phase D. The luteal-secretory phase E. The ovulatory phase

ANSWER: D The normal menstrual cycle can be divided into two segments: the ovarian cycle and the uterine cycle. The ovarian cycle is further divided into the follicular, ovulatory, and luteal phases. Similarly, the endometrium cycles through the menstrual, proliferative, and secretory phases, with the secretory phase corresponding to the ovulatory cycle's luteal phase. At the beginning of the menstrual cycle, during the ovarian follicular phase, gonadal hormone (estrogen and progesterone) and LH levels are low, and FSH levels begin to rise. This stimulates the growth of follicles that begin to secrete estrogen, which is the stimulus for the uterine proliferative phase. Just prior to ovulation there is an LH surge, accompanied by a smaller increase in FSH levels. After ovulation, there is a transition to the luteal-secretory phase, characterized by high estrogen and progesterone levels secreted by the corpus luteum, which in turn act centrally to suppress the pituitary release of gonadotropin hormones, resulting in low levels of FSH and LH. Serum progesterone determinations provide a reliable and objective measure of ovulatory function but must be performed at the appropriate time, which is usually a week before the expected onset of menses. A progesterone level >3 ng/mL is considered to be reliable evidence of recent ovulation.

You are seeing a patient for her Medicare annual wellness visit. Both she and her husband have had knee osteoarthritis for several years, and she has noted among her friends that men and women appear to receive different recommendations for osteoarthritis treatment by other physicians in the community. Which one of the following is true regarding osteoarthritis in women compared to osteoarthritis in men? A. The prevalence of osteoarthritis is lower in women B. Women tend to present with osteoarthritis at earlier stages than men C. Physicians are more likely to refer women for joint replacement surgery compared to men with the same degree of symptoms D. Outcomes from total joint arthroplasty are similar in women and men

ANSWER: D The overall prevalence of osteoarthritis (OA) of the hip, knee, and hands has been shown to be higher in women than men, with the differences largely attributable to an increasing incidence around menopause. The reason for this is likely multifactorial, related to differences in anatomic and physiologic factors. Women also tend to present with worse symptoms, including greater reports of pain and disability, and more gait changes. Differences in the prevalence of OA between men and women could also be explained by other factors, such as reduced volume of cartilage, bone loss, or lower muscle strength. Studies have shown that men are much more likely to receive a recommendation for knee replacement compared to women with the same presentation, even though women benefit from total joint replacement as much as men in terms of pain and functional outcomes.

A 25-year-old gravida 2 para 1 at 27 weeks gestation is brought to a rural emergency department on a spine board after a car she was a passenger in struck a tree at about 30 mph. The woman was in the front passenger seat and was wearing a lap and shoulder belt, and the front air bags deployed. She is awake and alert, and reports right shoulder and bilateral knee pain. She has not felt the baby move since the accident, about 40 minutes ago. Which one of the following is true regarding the management of this patient? A. Radiologic studies should be avoided B. Evaluation of fetal viability should occur prior to the maternal initial survey C. The mother should remain immobilized in the supine position until the cervical spine can be evaluated radiographically D. Fetal monitoring for a minimum of 4 hours is indicated E. Determination of Rh status is recommended only if she has vaginal bleeding

ANSWER: D Trauma occurs in 6%-7% of all pregnancies and is the leading cause of nonobstetric death. Motor vehicle accidents are the most common cause of trauma in pregnancy (48%-55%), followed by falls, assault, suicide, intimate partner violence, and homicide. Several maternal physiologic changes that occur during pregnancy make it more difficult to assess for maternal injury and hemorrhage. Plasma volume is increased by 30%-40%, making maternal blood pressure and pulse appear normal despite significant blood loss. Placental abruption and other complications of trauma can occur even after minor injuries. It is for this reason that electronic fetal monitoring is recommended for a minimum of 4 hours in any pregnant patient at ≥23 weeks gestation who experiences trauma and has a viable fetus (SOR B). The first step in the evaluation of a pregnant patient with trauma is assessing maternal stability with an initial survey of airway, breathing, and circulation. Once maternal respiratory and cardiovascular stability is ascertained, further evaluation of maternal injuries, as well as assessment of the fetus, can be undertaken. Supplemental oxygen; intravenous fluid such as normal saline or lactated Ringer's solution; and evaluation of maternal injuries, including radiologic studies with abdominal shielding, are all appropriate. Concerns about fetal exposure to radiation should not preclude or delay any needed radiologic evaluation. Radiographic studies that are routinely used in the evaluation of trauma during pregnancy represent a very low risk to the fetus. The workup of the pregnant patient should also include blood type and Rh factor, as uterine trauma has the potential to cause a break in the placental barrier, leading to fetal-maternal transfusion. If the mother is Rh negative and the fetus is Rh positive the maternal immune system may form anti-RhD antibodies. These antibodies cross the placenta and can lead to fetal hemolysis, anemia, hydrops fetalis, or even fetal death. All pregnant trauma patients who are Rh negative should receive Rh-immune globulin (RhoGAM) even if the trauma was minimal, as fetomaternal transfusion in amounts as small as 0.01-0.03 mL can cause maternal isoimmunization. The Kleihauer-Betke test is used to help determine if the quantity of fetal blood in the maternal bloodstream requires more than a single vial (300 µg) of RhoGAM to prevent maternal isoimmunization.

You are asked to develop an educational campaign to help women in your community understand the risk of heart disease and to help them identify and use appropriate preventive strategies. Which one of the following would be appropriate information to include in a program to educate women about heart disease? A. Smoking confers the same risk for coronary artery disease in women as in men B. Chest pain is uncommon in women with acute myocardial infarction C. Women are less likely than men to die as a result of their first myocardial infarction D. The American Heart Association recommends a minimum of 150 minutes per week of moderate-intensity physical activity for women E. Hormone therapy after menopause confers cardioprotective benefits in women

ANSWER: D Women and men differ in their risk for and presentation of heart disease. Women also respond differently to symptoms, as well as to prevention strategies and treatment. Further compounding this problem, many physicians have historically underestimated the burden of cardiac disease in women, which can further delay or hinder their care. Using a tool such as the Framingham risk calculator or the American College of Cardiology (ACC) risk calculator can help individualize risk-reduction strategies in both women and men. Risk levels can be calculated using age, sex, total- and HDL-cholesterol levels, smoking history, and blood pressure. Over a 10-year span, a coronary heart disease risk >20% is considered high risk, 10%-20% is intermediate risk, and <10% is low risk. "Optimal risk" is defined as optimal levels of all risk factors and adherence to a heart-healthy lifestyle (SOR A). While these tools are useful it is important to know that the ACC/American Heart Association risk scores were determined in predominantly male populations, and their thresholds may not as accurately predict risk in women. Although adult women smoke at a lower rate than men (15% versus 19%), a recent meta-analysis reported that in all age groups over 45 years, the additional risk for coronary artery disease conferred by cigarette smoking is 25% greater in women than in men. Furthermore, the combination of smoking with oral contraceptive use has a synergistic effect on the risk of acute myocardial infarction, stroke, and venous thromboembolism (SOR A). In addition, fewer women survive their first myocardial infarction compared to men. Women with heart disease have a higher prevalence of angina, a lower burden of obstructive coronary artery disease on angiography, and a poorer prognosis compared to men. Optimal recognition and timely management of acute myocardial infarction is critical. Although chest pain is the most common presenting symptom in both men and women, women were more likely than men to present without chest pain and have higher mortality than men, especially in younger age groups. Women should be advised to accumulate at least 150 minutes per week of moderate exercise, 75 minutes per week of vigorous exercise, or an equivalent combination of moderate- and vigorous-intensity aerobic physical activity. Aerobic activity should be performed in at least 10-minute increments, preferably spread throughout the week. Additional cardiovascular benefits are provided by increasing moderate-intensity aerobic physical activity to 300 minutes per week, vigorous-intensity physical activity to 150 minutes per week, or all activity to an equivalent combination of both. Observational data has shown an association between higher levels of physical activity and lower rates of many chronic diseases, including cardiovascular disease, as well as enhanced longevity. In addition, higher levels of activity were associated with commensurately lower rates of atherosclerotic cardiovascular disease in a curvilinear fashion (SOR B). Premenopausal women are relatively protected against cardiovascular disease compared with age-matched men. However, this sex gap narrows after menopause. A number of randomized, controlled trials, including the Women's Health Initiative, showed no cardioprotective benefit of postmenopausal estrogen use, and instead found an increased risk of both coronary heart disease and breast cancer, which led to sharp declines in the use of hormone replacement therapy (HRT). Similar results were seen in the Heart and Estrogen/progestin Replacement Study (HERS). For these reasons, HRT is not recommended for primary or secondary prevention of cardiovascular disease.

A 22-year-old female who is currently taking daily oral contraceptive pills would like to switch to a longer-acting contraceptive method. To avoid a gap in contraception, which one of the following would be most appropriate? A. If switching to medroxyprogesterone acetate (Depo-Provera), administer the first shot 3 days before stopping the pill B. If switching to the etonogestrel/ethinyl estradiol vaginal ring (NuvaRing), insert the ring 5 days before stopping the pill C. If switching to the norelgestromin/ethinyl estradiol transdermal patch (Xulane), start the patch the day after stopping the pill D. If switching to the etonogestrel implant (Nexplanon) or the levonorgestrel IUD (Mirena), insert the device on the day the pill is stopped E. If switching to the copper IUD (Paragard), the IUD can be inserted up to 5 days after stopping the pill

ANSWER: E Approximately one in five women (22.4%) change their contraceptive method within the first 6 months after initiation. Given that unintended pregnancy can occur during a gap in contraception, it is important to counsel patients about the transition and manage their switch effectively. The safest way to switch from one birth control to the next is by having no gaps and using condoms or spermicide for the first 7 days. The medroxyprogesterone acetate injection reaches full efficacy 7 days after initiation. Women switching to this contraceptive method from oral contraceptives should continue using the pill for 7 days. Hormones in the etonogestrel/ethinyl estradiol vaginal ring are rapidly absorbed, so women switching from an oral contraceptive should have the ring inserted one day before they take the last pill. Hormone levels take 48 hours to reach a plateau after a woman applies her first norelgestromin/ethinyl estradiol transdermal patch. When switching from a pill to a patch, a 2-day overlap prevents a decline in hormone levels, so women should start the patch the day before they take the last pill. Etonogestrel, the hormone in the subdermal implant, reaches peak serum levels in 4 days. Women who switch from oral contraceptives to the implant should continue taking the pill for 4 days after insertion of the implant. The levonorgestrel IUD takes 7 days to produce peak hormone concentrations. When switching from oral contraceptives, this IUD can be inserted up to 5 days after stopping the pill. The copper IUD becomes effective immediately upon insertion and also prevents pregnancy via spermicidal mechanisms up to 5 days after unprotected sexual intercourse. Similar to the levonorgestrel IUD, it can be inserted within 5 days of stopping the pill.

A 32-year-old female comes to your office because she has been unable to become pregnant after trying to conceive for 18 months. Her previous medical history includes mild endometriosis diagnosed by laparoscopy. Which one of the following treatments for endometriosis improves pregnancy rates? A. NSAIDs B. Oral contraceptive pills C. Androgenic hormones D. GnRH hormone analogues E. Surgical treatment of endometriosis lesions

ANSWER: E Endometriosis is a condition in which endometrial tissue is found outside the uterine cavity. It affects 6%-10% of reproductive-age women and 30%-50% of women with infertility. The mechanism by which endometriosis causes infertility is somewhat unclear. Several theories have been suggested, including distorted pelvic anatomy and tubal blockage, altered peritoneal function, and possible abnormalities in ovulatory and endocrine function. In women with mild to moderate endometriosis who are trying to conceive, surgical destruction of endometrial lesions has been shown to improve pregnancy rates (SOR A). Although NSAIDs (SOR C), oral contraceptive pills (SOR A), androgenic hormones such as danazol (SOR A), and ovulation suppression (SOR A) decrease pain in women with endometriosis, they do not increase pregnancy rates.

A 24-year-old primigravida at 22 weeks gestation sees you because of a depressed mood. For the last 6 weeks she has felt an increasing lack of energy and spends considerable time at home in bed because of this. She reports that she has been sleeping more than 12 hours a night for the last 2 weeks and has stopped her daily piano playing, which she usually enjoys. She has also noted difficulty concentrating. She indicates that family members are concerned about this change. She does not have any suicidal thoughts or feelings but does report feeling as if she has little hope of feeling better. The pregnancy has gone well up to this point. Which one of the following would be the most appropriate initial step in the management of this patient? A. Scheduling a visit in 4 weeks for follow-up and routine prenatal care B. Recommending an aerobic exercise program as the primary treatment C. Prescribing estrogen D. Prescribing paroxetine (Paxil) E. Referring her to a behavioral health specialist for cognitive behavioral therapy

ANSWER: E For women, the peak age of onset for depressive disorders is during the childbearing years. The criteria for diagnosing major depression in pregnancy are the same as for nonpregnant patients, and this patient meets several of the DSM-5 criteria for this diagnosis. Depression during pregnancy negatively affects maternal quality of life and is responsible for significant disability and health care utilization. It has been associated with increased risks of low birthweight, intrauterine growth restriction, preterm birth, and preeclampsia. There is also an association between prenatal depression and disruption in maternal-infant bonding, developmental delay, failure to thrive, and emotional and attachment problems. These have been shown to improve with treatment. Screening all pregnant women for depression has been shown to reduce symptoms in women with depression, and the American Academy of Pediatrics Mental Health Task Force recommends incorporating screening into the prenatal visit. Screening instruments can help identify women who need further evaluation and treatment, and it is important to use validated instruments because many of the symptoms associated with depression also occur with normal pregnancy. A preferred screening test has not been identified, but a two-step approach using the Patient Health Questionnaire-2 (PHQ-2) and PHQ-9 seems to work well. In women with a positive screening test for peripartum depression, cognitive behavioral therapy (CBT) has been shown to increase the likelihood of remission compared with usual care. SSRI use during pregnancy has been associated with a small increased risk of pulmonary hypertension and lower Apgar scores in newborns. However, prenatal use of paroxetine and fluoxetine has been associated with a small increase in birth defects. The association of paroxetine with cardiac defects led to its classification by the FDA as a category D agent during pregnancy. Pharmacotherapy should be considered in patients who do not have an adequate response to CBT. The medications with the best evidence of safety in pregnancy are sertraline, citalopram, and escitalopram, and no association has been demonstrated between severe birth defects and these agents.

A 23-year-old patient comes to your office 4 weeks after the uncomplicated vaginal birth of her first child, and reports that she feels tired all the time. On further questioning, she describes significant emotional lability during the first week after delivery. She has continued to have a low mood most days and worries about her ability to care for her child. She reports no personal or family history of depressive illness. Her infant is feeding and growing well, and now requires only one nighttime feeding. Which one of the following would be most appropriate at this point? A. Reassurance that the problem will most likely be resolved within 4 weeks B. Reassurance that this condition is unlikely to recur in subsequent pregnancies C. Appropriate screening for underlying medical conditions, including a urinalysis and an erythrocyte sedimentation rate D. Avoiding pharmacologic therapy because she is breastfeeding E. Recommending cognitive behavioral therapy together with pharmacologic approaches

ANSWER: E Postpartum depression is relatively common and occurs in up to one in seven women. Untreated, it is associated with significant maternal and neonatal mortality. It is disruptive to the family, and it can lead to a higher risk for paternal depression, marital discord, family violence, substance use and abuse, child abuse and neglect, failure to implement child safety and preventive measures, and poorer management of chronic health conditions in children. Postpartum depression is associated with both the early cessation of breastfeeding and reduced maternal-infant engagement, which can both have an adverse effect on infant development. Consequences of maternal depression include negative effects on cognitive development, social-emotional development, and behavior of the child. Sometimes it can be difficult to distinguish postpartum depression from "baby blues," a period of increased emotional lability, irritability, and fatigue that can begin in the first 24-48 hours post partum, has limited impact on functioning, and usually disappears within 2 weeks. Symptoms that persist beyond 2 weeks, including depressed mood, lack of pleasure, sleep disturbance, diminished concentration, feelings of guilt or worthlessness, loss of energy, or thoughts of death or suicide, are consistent with the DSM-5 diagnostic criteria for major depression. Women with a prior history of treated depression have a recurrence rate of more than 30% and these women should undergo preventive counseling as recommended by the U.S. Preventive Services Task Force (USPSTF). During the postpartum period, complications of pregnancy and common medical conditions can create symptoms similar to those of depression. Screening for anemia and thyroid disease is appropriate because they are often seen in the postpartum period. Screening tests should include a CBC (or hemoglobin or hematocrit) and a TSH level. All women should receive screening for postpartum depression where systems are in place to assure treatment and follow-up (SOR B). Additional questions about intimate partner violence, stressful life events, lack of social support, and financial stressors can help to assess the risk of depression early in the postpartum period. The American Academy of Pediatrics has published specific recommendations for screening mothers at the 1-, 2-, and 6-month well child visits, but this can also be accomplished at the 4- to 6-week postpartum visit. Either the Edinburgh Postnatal Depression Scale (EPDS), the Patient Health Questionnaire-2 (PHQ-2), or the PHQ-9 can be used for this screening. The EPDS is a 10-item instrument recommended by the USPSTF to screen for postpartum depression. It has been translated into multiple languages and can be downloaded at no cost. Multiple studies have confirmed its accuracy, and it can also aid in screening for postpartum major depression (SOR C). SSRIs have acceptable safety profiles and should be considered a mainstay in the treatment of moderate to severe postpartum depression. There is no evidence that one antidepressant is better than another in this group. Women who have previously responded well to an antidepressant agent should be initiated on that same medication post partum unless there is a known risk of harm to the infant (SOR C). For breastfeeding mothers, serum levels of sertraline, paroxetine, and nortriptyline have been shown to be undetectable in breast milk.


Set pelajaran terkait

The Heart: Superficial and Internal

View Set

Health Assessment: Rashid Ahmed Post-Simulation Quiz

View Set

Federal Income Tax (Individual) Exam 1

View Set

Ch. 21 - Ethnicity - Sherpath, Evolve, EOC

View Set

Ch. 2 Foundations of Wellness SG

View Set

Ch 15 Early Medieval Art in Europe

View Set

Chapter 29: Adolescent NCLEX style

View Set